Download as pdf
Download as pdf
You are on page 1of 547
Published by : LAXMI PUBLICATIONS (P) LTD. 22, Golden House, Daryaganj, New Delhi-110002. _ | 011-23 26 23 68 Phas | 011-23 26 23 70 Faxes; | 011-23 252572 011-23 26 22.79 Branches : ‘© 129/1, IlIrd Main Road, IX Cross, Chamrajpet, Bangalore (Phone : 080-26 61 15 61) © 26, Damodaran Street, T. Nagar, Chennai (Phone : 044-24 34 47 26) @ St. Benedict's Road, Cochin (Phone : 0484-239 70 04) © Pan Bazar, Rani Bari, Guwahati (Phones : 0361-254 36 69, 251 38 81) 4-2-453, Ist Floor, Ramkote, Hyderabad (Phone : 040-24 75 02 47) ¢ Adda Tanda, Chowk, N.D. 365, Jalandhar City (Phone : 0181-222 1272) © 106/A, Ist Floor, S.N. Banerjee Road, Kolkata (Phones : 033-22 27 37 73, 22 2752 47) 18, Madan Mohan Malviya Marg, Lucknow (Phone : 0522-220 95 78) | * 128A, Block 3, First Floor, Noorani Building, L.J. Road, Mumbai (Phone : 022-24 46 89 98) » Radha Govind Street, Tharpagna, Ranchi (Phone : 0651-230 77 64) EMAIL : colaxmi@hotmail.com WEBSITE : wwwlaxmipublications.com First Edition : July, 1965 Second Edition : Oct, 1967 Third Edition : dan, 1971 Fourth Edition : dan, 1974 Fifth Edition : May, 1978 Sixth Edition June, 1982 Seventh Edition : dan, 1986 Bighth Edition : Jen, 1989 Ninth Edition : April, 1992 Reprints : 1993, 1995 ‘Tenth Edition + Sept, 1996 Reprints : 1997, 1998 Eleventh Edition : Jan, 2000 Reprints + 2001, 2003 ‘Twolfth Edition : Aug, — 2004 Reprint : 2005 © 1971, 1989, 1992, 2000, 2004, B.C. PUNMIA © 1992, 2000, 2004, ASHOK KUMAR JAIN & ARUN KUMAR JAIN All Rights Reserved by the Authors. This book, or parts thereof, may not be reporudced in any form or translated without the written permission of the Authors. ‘ETS.0624.295. THEORY OF STRUCTURES Price : Rs. 228.00 Only —-10775/05/06 Typesetting by : Arihant Consultante, Jodhpur Printed at : Ajit Printers, Maujpur, Delhi-63 Contents NIEOR D ITED LOAD LONG N THE SPAN 0} RDER UNIFORMLY DISTRIBUTED LOAD SHORTER THAN THE SPAN OF THE GIRDER 8 3 INFLUENCE LINES FOR GIRDERS WITH FLOOR BEAMS INTRODUCTION P 8 INI F o LOAD POSITIONS FOR MAXIMUM $B INFLUENCE LINE OF BM. FOR GIRDER WITH FLOOR BEAMS a F cesee eee — é BBEREEBE: CHAPTER 8 THREE MOMENT EQUATION METHOD | | a | 3 © 4.12 BRACED CANTILEVER WITH SUSPENDED SPAN GIRDER CHAPTER 5 THE MULLER-BRESLAU PRINCIPLE 5. INTRODUCTION ” 52. THE MULLER-BRESLAU PRINCIPLE | ” .3.__INFLUENCE LINES FOR STATICALLY DETERMINATE BEAMS OR 54. PROPPED CANTILEVERS | 102 ss. CONTINUOUS BEAM : INFLUENCE LINE FOR BENDING MOMENT 104 82, A R 58 FIXED BEAMS 17 65.__DEGREE OF REDUNDANCY OF ARTICULATED sTRUcTURES | aa 66.__DEGREE OF REDUNDANCY OF RIGIDLY JOINTED FRAMES | 120 6.1.__METHODS OF ANALYSIS = at CHAPTER 7 THE GENERAL METHOD (METHOD OF CONSISTENT DEFORMATION) | NER D MAXWELL'S THEOREM : BETTIS RE &1.__INTRODUCTION : CONTINUOUS BEAM: 141 &2___CLAPEYRON’'S THEOREM OF THREE MOMENTS : DERIVATION 4 83.__CASE 1: EI CONSTANT : GENERAL LOADING as 86, -*CASE IV ; El CONSTANT ; SUPPORTS SETTLE : UDL. i) 87. VALUES OF FACTOR 8 OF #-DIAGRAM 144 &.8.___APPLICATION OF CLAPEYRON'S THEOREM FOR FIXED BEAM ___151. 89. ADDITIONAL, ILLUSTRATIVE. EXAMPLES [a4 CHAPTER oo oe ee cee 25, Ba. 135. 136. CHAPTER M4 @ 9 THE SLOPE DEFLECTION METHOD INTRODUCTION :_ SIGN CONVENTIONS FUNDAMENTAL EQUATIONS CONTINUOUS BEAMS AND FRAMES WITHOUT JOINT TRANSLATION PORTAL FRAMES WITH SIDE Way MINIMUM STRAIN ENERGY AND CASTIGLIANO'S SECOND THEOREM. ANALYSIS OF STATICALLY INDETERMINATE BEAMS AND PORTAL FRAMES BY MINIMUM STRAIN ENERGY 13. DEFLECTION OF PERFECT FRAMES GENERAL THE UNIT LOAD METHOD JOINT DEFLECTION IF LINEAR DEFORMATION OF ALL THE MEMBERS ARE KNOWN DEFLECTION BY CASTIGLIANO’S FIRST THEOREM MAXWELL'S RECIPROCAL THEOREM APPLIED TO FRAMES GRAPHICAL METHOD 14__ REDUNDANT FRAMES DEGREE OF REDUNDANCY ® ZREBE REBRER @RSBEERE KBRE eses 88 B REEE EF snes z 5. EEE ns 16. 167. 168. & E 1610. (ait) INTRODUCTION EQUILIBRIUM OF LIGHT CABLE “ABI UNIFORMLY LOADED CABLE ANCHOR CABLES TEMPERATURE STRESSES IN SUSPENSION CABLE THREE HINGED STIFFENING GIRDER TWO HINGED STIFFENING GIRDER TEMPERATURE STRESS IN TWO HINGED GIRDER TWO HINGED ARCH TWO HINGED PARABOLIC ARCH : EXPRESSION FOR H. TWO HINGED CIRCULAR ARCH : EXPRESSION FOR H MOVING LOADS ON TWO HINGED ARCHES ‘TEMPERATURE. EFFECTS 1611. REACTION LOCUS FOR TWO HINGED ARCH 1613. 16.12. FIXED ARCH THREE HINGED SPANDREL BRACED ARCH PART-II MISCELLANEOUS TOPICS a. 172. 173. 174. 175. 17___ WELDED JOINTS INTRODUCTION ADVANTAGES OF WELDING DISADVANTAGES OF WELDING TYPES OF WELDS AND WELDED JOINTS BUTT WELD OR GROOVE WELD BeBBEB SSBSRRee EGRGBSRSERERE 431 453 453 116. 477. 178. ai FILLET WELDS DEFECTS IN WELDING WORKING STRESSES IN WELDS DESIGN OF FILLET WELDS FOR AXIAL LOADS HILLET_WELDING OF _UNSYMMETRICAL SECTIONS : AXIAL__LOAD. DESIGN OF BUTT WELDS DESIGN OF PLUG AND SLOT WELDS ECCENTRICALLY LOADED FILLET WELDED JOINTS ECCENTRICALLY LOADED BUTT WELDED JOINTS 18___METHOD OF TENSION COEFFICIENTS INTRODUCTION ‘TENSION COEFFICIENTS. 20.13. 2.14. CHAPTER aa. m2. a3. aa. as. 20 ‘PLASTIC +THEORY INTRODUCTION ‘THE DUCTILITY OF STEEL ULTIMATE LOAD CARRYING CAPACITY OF MEMBERS CARRYING AXIAL TENSION PLASTIC BENDING OF BEAMS STAGES OF BENDING OF RECTANGULAR SECTIONS. EVALUATION OF FULLY PLASTIC MOMENT EVALUATION OF SHAPE FACTOR MOMENT-CURVATURE RELATIONSHIPS PLASTIC HINGE, LOAD FACTOR CONDITIONS AND BASIC THEOREMS OF PLASTIC ANALYSIS DETERMINATION OF COLLAPSE LOAD FOR SOME STANDARD CASES OF BEAMS PORTAL FRAMES DESIGN RECOMMENDATIONS 21 BUILDING FRAMES INTRODUCTION : BUILDING FRAMES BRACING OF MULTISTOREY BUILDING FRAMES DIAPHRAGMS, SHEAR WALLS OR CORES TUBE STRUCTURES SUBSTITUTE FRAMES 456 437 458 838 461 470 eee s8B Seees a8 1 BEB eee $58 559 561 oO) 216. ANALYSIS FOR VERTICAL LOADS | 565 21.7. METHODS OF COMPUTING BM. 566 218 ANALYSIS OF FRAMES SUBJECTED TO HORIZONTAL FORCES S73 219. PORTAL METHOD | 314 21.10. CANTILEVER METHOD | 575 21.11. FACTOR METHOD $81 CHAPTER 22 __KANP’S METHOD 22..__INTRODUCTION 51 222__CONTINUOUS BEAMS AND FRAMES WITHOUT JOINT TRANSLATION | ___591 223. SYMMETRICAL FRAMES 607 22.4. FRAMES WITH SIDESWAY 609 CHAPTER 23 UNSYMMETRICAL BENDING 23.1. INTRODUCTION | 623 232. CENTROIDAL PRINCIPAL AXES OF A SECTION 623 23.3. GRAPHICAL METHOD FOR LOCATING PRINCIPAL AXES __625 234. MOMENTS OF INERTIA REFERRED TO ANY SET OF RECTANGULAR AXES 88 238. LOCATION OF NEUTRAL AXIS 239. GRAPHICAL METHOD : MOMENTAL ELLIPSE 73.10._THE_Z-POLYGON 2311. DEFLECTION OF BEAM UNDER UNSYMMETRICAL BENDING 639, INDEX aT agape PART-1 MOVING LOADS ROLLING LOADS INFLUENCE LINES INFLUENCE LINES FOR GIRDERS WITH FLOOR BEAMS INFLUENCE LINES FOR STRESSES IN FRAMES THE MULLER-BRESLAU PRINCIPLE ci pyrighted material Rolling Loads 1.1. INTRODUCTION In the case of static or fixed load positions, the BM. and SF. diagrams can be plotted for a girder, by the simple principles of statics. In the case of rolling loads, however, the B.M. and S.F. at a section of the girder change as the loads move from one position to the other. The problem is, therefore, two-fold : ( to determine the load positions for maximum bending moment or shear force for a given section of a girder and to compute its value, and (ji) to determine the load positions so as to cause absolute maximum bending moment or shear force anywhere on the girder. For every cross-section of girder, the maximum B.M. and S.F. can be worked out by placing the loads in appropriate positions. When these are plotted for all the sections of the girder, we get the maximum BM. and maximum S.F. diagrams. The ordinate of a maximum B.M. or SF. diagram at a section gives the maximum B.M. (or SF.) at that section, due to a given train of loads. We shall consider the following cases of loadings = Single concentrated load. Uniformly distributed load longer than the span of the girder. Uniformly distributed load shorter than the span of the girder. Two loads with a specified distance between them. Multiple concentrated loads (train of wheel loads). Sign Conventions ‘The following sign conventions will be followed for B.M. and SF. at a given section (Fig. 1.1). (1) A shear force having an upward I direction to the left hand side of a section ‘or downwards to the right of the section will be taken positive. Similarly, a nega- tive S.F. will be one that has a downward direction to the left of the section or | +VESF -VESF +VEBM -VEBM @ eo) (e) @ FIG. 1.1 @ 4 ‘THEORY OF STRUCTURES upward direction to the right of the section [Fig. 1.1(a, 5)]. (2) A B.M. causing concavity upwards will be taken as positive and will be called sagging BM. Similarly a B.M. causing convexity upwards will be taken as negative, and will be called hogging bending moment [Fig. 1.1(d)]. 12. SINGLE CONCENTRATED LOAD Let us now consider a single concentrated load W travelling or rolling along a simply supported beam or girder AB, of span L, from left to right. (a) MAXIMUM SHEAR FORCE DIAGRAMS Consider a point C, distant x from the left support A. Let the distance of load W be y from A, For any load position, the reaction Ry= and R= Wenn (ai) Load in-AC (y x) FIG. 12, ‘We have seen that when the wheel load W reaches the section C, maximum —ve S.F. of value ~ 7 occurs at the section, When the load moves further (ce. when y becomes greater than x), we have Fx (at ©) = + a= +¥e—y 2) Thus, the shear force changes sign immediately when the load crosses the section. The maximum positive S.F. occurs evidently when y =x (ie. when y is least and the load is in CB). ROLLING LOADS 5 ‘Thus, Fax = + Me (1.2) For different values of x (ce. for different positions of the section C), the maximum positive shear force, given by equation 1.2 will vary linearly with x. Thus, atx =0, Fae (+) =+ + We Frnce mex at x=L, Frac (+) = + HED o, The absolute maximum +ve SF, therefore, occurs at the left hand support, its value being + W. The maximum positive S.FD. is represented by aa:b of Fig. 1.2 (6). (0) MAXIMUM BENDING MOMENT DIAGRAM Let us now draw the maximum bending moment diagram for the beam AB. It must be noted that a simply supported beam, under downward loads, bends causing concavity to the upper side. Hence the bending moment is always positive for all sections of the beam. Therefore, the maximum bending moment diagram will also be positive. (bi) Load in AC (y x) When the load W crosses the section C, My =+Ry.x= + 2E=Y, 2 ‘This increases as y decreases. When the load is on the section C, y =x, and the section has*the maximum bending moment, Mme = 1 UGeas which is the same as equation 1.3. Thus, the maximum bending moment at a section occurs when the load is on the section itself. For different values of x (ie. for different positions of section C) the maximum bending moment given by equation 13 will vary parabolically with x. Fig. 1.2(c) shows the maximum bending moment diagram. For absolute maximum bending moment dM mae _ . Wa apn ab OE =0 + EE-2x)=0 or x= Thus, the absolute maximum bending moment occurs at the centre of the span, and its value is given by 6 ‘THEORY OF STRUCTURES 13. UNIFORMLY DISTRIBUTED LOAD LONGER THAN THESPAN OF THE GIRDER Let us now study the case of the uniformly distributed load w per unit length, longer than the span, and moving from left to right. (@) MAXIMUM S.F. DIAGRAM Let us consider a section C distant x from left support A, as shown in Fig: 1.3(a). Let the head of the load be distant y from A. Reaction Ru =f When the load is in AC (ie. y < 2), Fx =-Re (1) This evidently increases as y increases, until the head of the load reaches the section C (ie, when y=x ). we Frmax = — (1.4) When the load still moves further, it can be proved that the value of Fx given by equation 1.4 decreases. To prove this statement, let the head of the load move by a distance dx from CC towards B, and let dRp be the corresponding increase in the reaction at B. 2 ‘Then Ry + Ra =etey 2 and Fg =~ (Ry + AR) tw dre — EAN to wa! or anay - (7-9) ~@) (neglecting the small quantities of sceond order). Since [is less than 1, the expression inside the bracket is negative. Hence Fx given by (2) is less than Fx given by equation 1.4. Thus, the maximum negative shear at a section occurs when the head of the load reaches the section, (i.e. when the left-portion AC is loaded and the WIM i a —— The maximum negative S.F. L- diagram can be plotted by giving 3 different values of x in equation vf Po Thus, at x=0, Frac wl? wm 2 Atx=L, Fina (— (0) Max. S.F.D. by The absolute maximum nega- tive SF. occurs at the right hand support. The maximum —ve SED. is shown by abb; in Fig. 13(6). (o) Max. BM.0. FIG. 13, ROLLING LOADS: 7 The S.F. at section C will continue to decrease as the load advances further. When the load covers the entire span, Fe + Ra — wnt BE A (3) This is positive for the section C to, de in the left half of the portion, Let the load still move on so that the portion CB is fully loaded and portion AC is partially loaded, and we have Fx =— Rp + w(L —x) (4) In the above expression, the quantity w(L —x) remains constant as the load still moves further, while Ra diminishes. Thus, with the onward movement of the load, the positive value of Fx increases. When the tail of the load reaches the section C, we have Fy =+ Raz wi =x) This is the maximum value of positive shear force at C. As the load moves further, Ra decreases, and hence Fy decreases. > Thus, Fmae =+ ven (15) The maximum positive shear force thus occurs when AC is empty and CB is fully loaded. To plot the maximum positive SF. diagram vary x from 0 to L. + ME Fnoe me (+¥6) wl At x=0, Fmax =+ Fe 3 Atx=L, Fran =0 The maximum +ve S.FD. is shown by aa,b in Fig. 1.3 (6). (6) MAXIMUM B.M. DIAGRAM 2 Let the head of the load be in AC, such thaty x). ae Ree nar =— Ro=— 5 (1.62) This is a parabolic relation and is valid for values of x between 0 and a. At 2=0, Free= 0. and atx =a, Fray 2 (2) Let the position of the section C be such that x >a. When the load is in AC, the portion AC is partially loaded, and F.=— Re, which goes on increasing as the head of the load approaches C. When the head of the load reaches C, we have Fax =— Ro=— 2 This is a straight line relation. me eee S A B x——t (a) The beam (0) Max. S.F.D. FIG. 14. ROLLINGLOADS 9 Al x= At x=a, AtxsL The maximum —ve S.FD. thus consists of a parabola upto a distance of a from 4, and then straight line upto B. (ait) Maximum Positive S.F. ‘As the load moves further, the S.F. decreases. For a particular load position, it becomes zero, and then changes sign and becomes positive. As the load still moves further, the positive SF. at C increases. For maximum positive shear force at C, the span AC should be empty and the reaction at A should be a maximum. In other words, the tail of the load should be at C, and the load should extend from C towards B. When the tail of the load is at C, wa a Faz =+ Ret “(Lx - 5) (17) This is the equation of a feign line, and is valid for all values of x between 0 to (L ~ a). Thus, it x=0, Fre = + ™ T(e-$) > = (L-" at ZZ (b= li ao At x= (L~a), Fax =+ 2 (L-L+a-S\=+57. When the position of the section C is such that x>L—a ie. when x varies from (L ~a) to L} Fra, = + Rat x x (1.8) Thus, Fmae is independent of a, and varies parabolically. ae Atx=L—a, Fou, at P-L tant as before At x=L, Thus, the maximum positive shear force diagram is a straight line from x=0 to x=L—a, and a parabola between x=L—a to x1=L. The absolute maximum negative S.F. occurs at support B, when the head of the load is at B, and the absolute maximum positive S.F. is at A when the tail of the load is at A. The maximum negative and positive S.F. diagrams have been shown in Fig. 1.4(6). (6) MAXIMUM B.M. DIAGRAM Let the length of the U.D.L. be a. When the load is in the portion AC, the BM. at the section C is given by Mx =+ Re (L -x) This goes on increasing as the head of the load approaches the section C. When the head of the load crosses the section C, the B.M. still goes on increasing, till it attains maximum value at a specific load position. On further movement, the B.M. at the section C decreases. 10 ‘THEORY OF STRUCTURES For the maximum B.M. at the section aad yin Ae tom ae AAS St, _s 5(a). o_o In this load position, Ro = way a (a) The beam Distance CB, = (7 =r +4) Mx =+ Ro (L —x) - wi? (b) Max. B.M.D. For Mx to be maximum, differentiate it FIG. 15. with respect to y and equate to zero. Thus, we have of s0=+ BL -x)-w(y-1 +9) or FL-x) y-x+$) _ 9) In the above equation, a= 4,B; ; L=AB ; L -x=CB and yout ; =CB, Hence equation 1.9 can be expressed geometrically as ABs ca = ca, or SB pe AB, ABS CB AC. CB AB AB CB, AC or AC AC (1.10) Ch CB Thus, for maximum bending moment at a section, the load position is such that the section divides the load in the same ratio as it divides the span. This relation will be found to hold good generally, both for the point loads as well as the uniformly distributed loads. Equation 1.9 is directly useful for the location of the U.D.L. for the maximum B. For the maximum B.M. at C, we get, from equation 1.9, =2qL- ~8n% x HEE -n tr Fa t2-F a and yrxt$ FL-4 Substituting the values in (2), we get Mme =+ 5 (L-x)(Ste-F L! +Fe- -9 (1-37) | (dL) ROLLING LOADS un ‘The maximum B.M. diagram can now be plotted by giving different values to x in equation 1.11. Absolute maximum B.M. occurs evidently at the centre, when x =L/2. Thus, from equation 1.11, Macnee =+ "#4 (1-4) (1-2) a4 Mie 4) The above value can also be obtained by considering Fig. 1.5, and applying the deduction of equation 1.10 independently. Thus, for maximum bending moment at the centre of the span, AC _AC CB CB where AC =cB=% AC LLL, L/2 or =CB=a/2 In this position, Ra = Ro= “4 and = Mmax (al centre) = Mmuxmax=+ which is same as before. 1.5. TWO POINT LOADS WITH A FIXED DISTANCE BETWEEN THEM Let us now consider two point loads, W; and W: al a fixed distance d apart, moving from left to right with W; leading. Let the leading load W be smaller than 7. (a) MAXIMUM NEGATIVE SHEAR FORCE For negative shear force at the section C, we have to consider the three load positions: (1) Both loads to the left of the section C. (2) Load W, to the right of C and HW, to the left of it. (3) Both loads to the right of C : For this load position, there will be no negative shear (Since Fx=+ Ra), and hence we will consider only the first two load positions. (ai) Both Loads to the Left of C For this load position, Fx =-Rs i) This increases as the leading load reaches near the section C, and is maximum when W, is just 10 the left of C. (1) When x < d, only W; will be on the girder and W2 will be off the span, with W, at C. Hence, Fmax =~ Ra= L sne(I) -2(1-12) (2) When x >d, both W; and W, will be on the girder with W, at C, Hence Wixt He @ wn (D) (1.13), 2 THEORY OF STRUCTURES (aii) W, to the Right of C and W, to the Left of C For this load_ position, Fx =- Rot WH @) Since Rp increases as W, and W; reach near B, the maximum S.F. occurs when the load W, is just to the left of C. (8) When (L - x) > d, both W, and W, will be on the beam with W, at C, Hence "Fax =— Ra + Wr -_ Basti (td) m nH) ...(114) (4) When (L — x) Mxt Wierd) yy, ROLLING LOADS 3 WL -d) or sca (1.16) Thus, when x < ne, equation I will give greater Fa: A Beyond this value {ie eo ead tors ) equation III will give greater Frev. h () Zone Q) [x=d tox=L-d] The second zone under consideration is from x =d to x=L —d, and for this both equations II as well as IMI will be applicable. For Eq. (Il), W; is at C and W, is to the left of it. For Eq. (10), W3 is at C and W is to the right of it. Out of the two, equation (Il) will give larger value if Fo > Fas ie WttWe-d > Wet MHD o | WM 4Wd ML L or a ——_, equation Ill will give Fas: and maximum —ve S.F. aw? Cation I will give ani r will occur when the rear wheel load reaches the section. Thus from Eq. l, when x= 0, Faux From Eq. II when x=d, Fuss (0) MAXIMUM POSITIVE SHEAR FORCE In this case also, we will consider the three load positions for maximum positive shear force at the section (C) (1) Both loads to the right of C. (2) W to the right of C and W; to the left of C. (3) Both loads to the left of C : For this position, there will be no positive S.F. (Since Rp) and hence we will consider only the first two load positions. @ Both loads to the right of C For this load position, Fx =+ Ra ~() This increases when Ra increases. Hence the maximum value occurs when W; is just to the right of C. F= 4 ‘THEORY OF STRUCTURES (1) When (L—x)>d, both W, and W, will be on the girder, with W, at C. Hence Fax =t Ra= + a (V) ...(1.18) (2) When (Lx) d, both W; and W2 will be on the girder with W just to the: right of Cc. Hence Fag = MLD HM 8 Hy, Equations V, V1, VII and VIII are valid for appropriate ranges of x. For the case when W; < W:, equation V will give the maximum positive shear force, and is valid for all values of x between 0 to (L—d). Beyond this, W; is off the girder, and equation VI will be valid. Thus, at r= 0, Fax =+ (Wi + WEED) Wid L The complete maximum S.F.D. has been shown in Fig. 1.6 (b). Eqs. Wil and VII will give maximum value only when W; Mx > ‘Mx It is clear, therefore, that in either case, *Mx will not be maximum. Hence maximum B.M. at the section is either 'Mx or “Mx, whichever is larger. Fig. 1.6 (© shows both the parabolas giving ‘Mx and *Mx governed by equations | and II respectively. Now 'My is greater than "Mx if WxtMeE-d _ me y+ ml-2 Tow» > 2 x ie. x> Ab on(1.24) W+W, For x < AL yy is: maximum W+W, For x > WL My is ‘maximum. W+W: Now, 'Mx is zero atx = 24, and at x =L. W+ Ww, Wid *Mx is zero atx =0, and at (L-x)= Ww, 16 ‘THEORY OF STRUCTURES Both the parabolas cross each other at F ', where 'Mx=Mx. To find the position of this section, put 'My= ’Mx. Thus Ws ted yy Mb -2-d) + Wi(L — b L WL WtW ‘Thus, F divides AB in the ratio of W:W,, For all sections from A to F, maximum B.M. is given by ‘My, and for all sections from F to B, the maximum B.M. is given by ‘Mx. The maximum value of “Mx in equation II will occur at aE Ap Wie 2°52 ( Wt | ‘When W; > W;, then absolutely maximum B.M. anywhere in the girder occurs in the “Mx range at x=} AE. From which x (or AF) = x In case (L—x)9.6 m, 'Mx is greater than *Mx. Since 9.6 m, for x<9.6 m, "Mx is greater than ‘Mx, and for At x=96 m, “My =+ 38 (136 — 96)=+ 24 kN-m My, = (96 — 36) (1-52 )=+ 24 kNm Thus “Mx and ‘Mx are equal at x= 9.6 m (This is a check). ROLLING LOADS a (©) Max. BM. under W2 when Wyis oe girder ("Mx) From equation 125, “My =+ MEL ~2y=+ $2 a6-1 and this is valid from x = L -d= ie 10 m, to r = L= 16 m. In this range "Mx is greater than 'Mx if ~ 6-2) > (36-10) (22) or if 6x >10x-36 or aon or ox<9Om But since the equation of “Mx is valid for x greater than 10 m, the above condition cannot be fulfilled, and hence "My is less than 'Mx between x= 10 to x= 16 m. “Mx has its maximum value at x= L/2=8 m, its value being equal to My =+ #8 a6 - 8)=+24 kN-m The maximum BMD. has been drawn in Fig. 18(c). Example 1.3. Solve example 1.2 if Wi=4 KN ; Wy = 6m and the span L=12 m. Solution ; For the present case 1 _y, = 4% 124g m W.+W, 4+6 a=6> ul Wm Thus the case is not the standard one, and the S.ED. will not be similar to that of Fig. 1.6(6). (@ Maximum - ve S.F. Diagram @ For x= 0 to x=d=6 m, when only ™ is on the span, and W; is off the span the maximum S.F., from equation 1.12, is given by ea Mit Ate R z 773 KN wa) (i) For x=0 to x=d = 6, when W; is just to the left of the section and W; ahead of ‘Fras it. Fax =— Ry + Wy Wax Wie +d) 1G29 =( Spo Feat Mm Se +45 (3 2) wa (2) Thus, for x=0 to x=6 m, Fmax is given by both equation (1) and (2). *Fmax will be greater than ‘Fax only if (2+-2)>% or if Sxr-12 >2x or if 3x >12 or if x>4 Thus, for x=0 to x=4 m, ‘Fmm (given by equation 1) gives the maximum S.F. while for x=4 to x=6 m, *Fmax gives the maximum values. 2 THEORY OF STRUCTURES Thus, atre4m, Fi =-} kN al x=6 m, Fy =-3 kN (ii) For s=d=6 m (or L-d=6 m) t x=L=12 : both W, and W, on the span : The maximum S.F. is given by equation 1.13 tp ae Row zt Mae — a) Fam =— Ro= z ew AHO ESO) 85 a. D Ze-39) (3) (@) For r=d=6 m to x=L=12, with W; on the section and W; off the girder : ‘The maximum S.F. is given by equation 1.5. Fux =— Ro! ) Thus, for x =6 to x= 12 m, the maximum S.F- is given by equation (3) and (4). Evidently “Fey Will be greater than ‘Fa if ros p> ge-39) oF x<9 Thus, from x=6 to x=9 m, maximum S.F. will be governed by "Fux (equation 4), while from x=9 to x= 12 m, maximum SF. will be governed by ‘Fux - m, Fy =-45 KN 2 m, Fu =-7 kN (©) Maximum Positive S.F. Diagram () When both W, and Wy are on the girder : Max. + ve S.F. at C will occur when W; is just to the right of C and W is ahead of it by 6m. Thus, from equation 1.18, Fag <4 Rot BEDE 2d a4 SDF 4U2-F-9_ 4 (gS ib _, 6x2 Wi+W, 4+6 Thus for x=0 to x= 7.2 m, max. B.M. will be governed by "Mma and from x>7.2 m, max. B.M. may be governed by Mine >7.2 m. u THEORY OF STRUCTURES At x=7.2 m, “Mia =+(2x 72-3) (12-7.2)= +14.4 kN-m "Mig = +72(8 3 (iii) Max. bending moment under W, with W, off the girder my, a+ Wat ‘Mx =+ > x 72)= + 14.4 kKN-m (check) (L-x)=+ S£a2 =x=+x(6-05x) (Eq. 1.25) ...(1) To get the section where *Mx is cqual to “My, we have or ox=6m The common value of B.M. is given by "Ms = "Mo=+ 6 (6-05 x 6)=+ 18 kKN-m To get the section where "Mx and 'Mx are equal, we have (s*- ‘The common value of the max. B.M. is given by "My = 'Mo=t 9 (6-05 x 9)=+135 kN-m 3) (12-2) =x(6-05x), which gives x= 9 m. The maximum value of "My evidently occurs at x= & "M.=+ 6(6 — 05 x6)=+ 18 kN-m Hence, to Summarise : (i) For r=0 to x=6 m, Max. BM. is governed by "Mx. (ii) For x=6 m to x=9 m, Max. BM. is governed by "Mx. (i) For x=9 m to x= 12 m, Max. B.M. is governed by ‘Mx. The complete Max. B.M. diagram is shown in Fig. 1.9(c). The absolute Max. BM. will be under W;, at x= 4.8 m, its value being equal to 19.2 kN-m. = 6 m its value being equal to Example 1.4. Two point loads, W, and W,(W:>W,) spaced at a distance ‘d’ wravel from left to right across a simply supported girder, with W, leading. Prove that the limiting span below which the greatest bending momeni anywhere in the girder will occur when the load W, has gone Off the girder, is equal to {r= Hence, draw the max. B.M. diagram if Wi=4kN ; W:= 6KN ; d= 6 m and the span L = 10 m. Solution : (Refer Fig. 16 ¢ d, e). Since W; is greater than W,, the maximum value of the BM. ‘Mx at a section occurs when the load W; is at the section, with W; ahead of it. The maximum value of ‘Max Occurs _4E 1 Wid at r= L-———— WietWw, 2 } The maximum value of *fnax is obtained by substituting the value of x in equation 1.23. ) ROLLING LOADS 2s Thus "Mnax =7 [M@€-x-d) +Hb-y] 1 Wd 1 ad a5 "4 law, +o, arg -W, lt mam|[ [+2 [arm 1 = _ | | (i+ WL - Wd W, + W2) L — Wid! qm lt Hat ae widl] 2 = mt WL — Wray (2) sn(1.27) H+) Also, when W; its off the girder, the maximum B.M. under W; at the section is given by equation 1.25, wy, = Wty Mz =P (Lx). Evidently, its maximum value occurs at x= Wak ( ~4)=-mb @ nit "Ma = Lx2 2 To have the greatest BM. governed by “Mmar, we have WAL . (M+ WL - Map 4 aL (Wit Wr or WLW, + Wr) > (Wi + WiyL? + W} a — 20, dL (W, + Wr) or LW, + W,) (M+ W) — Wa] + Wid? — 2d (MW, + Wy) <0 or L? (W, + W,) - 2d (W, + Ws) + Wd<0 which gives iis 1-—_% la Wit Wy or L(09x — 36) (10 —x) or if, 06x >0.9x - 3.6 or if x<12 m. However, since maximum value of = 10 m, “Mx will always be greater than ‘My, The absolute maximum B.M. any where in the girder will evidently be governed by "Mx. It will occur at x= L/2=5 m and its value is Mmocmac ="Ms=+ 0.6 x 5x 5=+ 15 kN-m. ‘The maximum B.MD. is shown in Fig. 111. FIG. 1.11. 8 ‘THEORY OF STRUCTURES 1.6. SEVERAL POINT LOADS : MAXIMUM B.M. Let us now take the case of a train of wheel loads W;, W2..Wq crossing a simply supported girder. For getting the position and amount of maximum bending moment, we shall discuss the following two propositions : PROPOSITION 1 When a series of wheel-loads cross a girder, simply supported at the ends, the maximum bending moment under any given wheel load occurs when the centre of the span is mid way between the CG. of the load system and the wheel load under consideration. ‘Thus, in Fig. 1.12, let us find the max- imum bending moment under the wheel load W, of the train of wheel loads Wi, Wy ...W,. Let W, be the resultant of all loads to the left of Ws and Wx be the resultant of all loads to the right of W; and inclusive of Ws. Let W be the resultant of the load system, situated at a from 1, b from We and c from Ws, For given load system, a, b and c are constants. To get the maximum B.M. under Ws, let the load W, be placed at a distance 7 from the centre C of the span. It is required to find the value of the variable z. Wee Reaction R=7[F+e-2| B.M. under Ws is M wes (rl -WMate | FIG. 1.12, =+ 2 E+¢e-2)G+)-meto wee -2 +E) mate) ; dM _,W _ Le For maximum M, M4 ¥(e—2)=0 or z=§ (1.29) Hence the centre of the span is midway between W and W;, This proves the proposition. The above proposition can be used to find the maximum BM. under desired wheel load. However, to get absolute maximum B.M, any where on the girder, several trials) are to be made. Any one load must first be chosen and arranged according to the condition of equation 1.29 derived above, and the maximum B.M. is calculated. Another wheel load can then be chosen and the procedure repeated to get another value of maximum BM. Two or three such trials may sometimes be needed, and the absolute maximum B.M. will be the greatest of these. However, to reduce the number of trials to a minimum, the following points must always be kept in mind : 1, The maximum B.M. always occurs under 2 wheel load, and not any where between two wheel loads. 2. Absolute maximum B.M. always occurs at a section near the centre of the span. (It never occurs at the centre unless the C.G. of the resultant load coincides with the centre line of some heavy wheel load). ROLLING LOADS 29 3. The wheel load should be so selected that the centre of the span is midway between the C.G. of the load system and wheel load under consideration. 4. The absolute maximum B.M. generally occurs under the heavier wheel load - specially that which is very near to the CG. of the the load system. PROPOSITION 2 The maximum bending moment at any given section of a simply supported beam, due to given system of point loads crossing the beam occurs when the average loading on the portion to the left of it is equal to the average loading to the right of it, ie. when the section divides the load in the same ratio as it divides the_span This proposition is very useful for locat- ing the load position for maximum B.M. at a given section, and has already been proved for uniformly distributed load in § 14. Let it be required to find the load position for maximum B.M. at a point C, distant x from A. Let W be the resultant load located at y from A, for maximum BM. at C. Let W be the resultant of the loads to the left of C and Wx the resultant FIG. 113 to the right of C. ny Win My =+ Ry. -Wibe-(- aj-+¥E-Y, -Wa@-yta). In the above expression, y is the only variable. For maximum Mx, dy __ WX 4 y,=0 (or should change sign) 8 wm _ We x L L-x L-x In other words, the average load on the portion to the left of C is equal to the average (1.30) load on the portion to the right of C. Actually, in isolated load systems, ra cannot be equal to zero but will have sudden steps changing from a positive value to a negative one, Generally, the loading will be such that either 4C is heavier and CB lighter, or vice versa. Hence the maximum B.M. at C will occur when { Wes. x L-x Wa and decreasing W.. Hence to get the value of maximum BM. at a section, one of the wheel loads should be placed at the section, so that if that load is considered as a part of W., the We r= (We _ _We ) becomes negative. If on rolling the loads from left to right, (—*—7—*-| does ) changes sign. The vaiue } can change sign only when a load crosses C from left to right, thus increasing " wy ¥ 1 i “ expression (=- S) is positive, but if considered as part of Wa, the expression (& soi x Lex % ‘THEORY OF STRUCTURES not change the sign from +ve to —ve, but instead, increases or remains positive, the loads should be rolled to the right so that next load comes over. to the section. With this new load at the section, (ue - 7) should again be investigated for the two positions, aS described above, till it changes sign. In the passage of a series of wheel loads, two or more positions of the load system may occur satisfying the above condition of change of sign from +ve to —ve. In such a case, the value of Mx at the section for each of these load positions must be calculated, and the greatest of these taken as the maximum BM. at the section. It must always be remembered that maximum B.M. at any section occurs when the wheel load is over it. 1.7, SEVERAL POINT LOADS : MAX. S.F, AT A SECTION: Let us now investigate the load position for getting maximum S.F. at a section due to several point loads 1, W..., Wa. The process of locating the load position for maxima is that of trial and error. However, the max. SF, at the section occurs when one of the loads is on the section. We Wy W, To get the max. —ve S.F. at C let the load WM be at the section CC, and let another load W; be at d behind it. If the loads are rolled to the right by a distance d, so that W, comes at C, the S.F. at the section C will be changed. This change (SF) consists of two components : () Increase dR» (gradually as the loads roll) AG. 1.14. ORy = 44, where W= resultant of all loads on the span. (ii) Sudden decrease or drop equal 10 Hence OF =8Rn-M% me -W, (31) If this change is positive, rolling will increase — ve S.F. In such a case, the rolling must be contained till equation 1.31 becomes negative. The above discussion is true only if no load either enters or leaves the span when the system is rolled by the specified distance d. To discuss the most general case, let the load Q enter the span by a distance a, and load P move beyond B by a distance b, duc to rolling, If W is the resultant load before the advance, we have | — Wa , Qa_ 4 ORy = + SE Pitz} =6R, —-W = 44 4 Qe - Hence OF = Ry — Mix H+ HP(L+ Tm wa 7 Qa apr etm- 7+ (1.32) ROLLING LOADS 3 Since : and ¢ are usually small compared with unity, the last two terms of the above expression May be neglected for approximation. Hence, we get é, ar =H _(H,+ P) (1.33) If this is + ve, rolling will increase the S.F. From the above, it is evident that the load entering the span does not change the S.F. appreciably while the load leaving the span does. If all the loads are equal and equally spaced { Ma —W,) will always be negative and, hence, maximum S.F. at the section will occur when the first load reaches the section. The absolute maximum —ve S.P. evidently occurs at the right support, for which the criterion of equation 1.33 must be tied. Equation 1.31 (or 1.33) can also be used for getting maximum +ve S.F. at the section. However, in this case, the advance (or rolling) must be to the left till 6F is increaséd (or becomes + ve). The first chosen wheel load W; is considered just to the right of the section before such movement. If the whole system is now moved to the left by distance, say c, positive SF. will increase only if (42 -m) is positive. If it becomes negative, the load position before such movement gives maximum +ve S.F. If it becomes positive, movement must be permitted till the expression becomes negative. Example 1.6. The system of concentrated loads shown in Fig. 1.15 (a) rolis from left to right across a beam simply supported over a span of 40 m, the 4 KN load leading. For a section 15 m from the left hand support, determine: (a) The maximum bending moment. 6 6 10 10 4KN (b) The maximum shearing force. tetetetet > Solution : 8 2 28 (@) Maximum BM. wi By inspection, it is clear that the 40 m maximum BM. at C will occur when the central 10 KN load is over the section, & 8 1010 44N so that when the loads are rolled across A the section, the condition of loading in AC ~ CB will alter from heavier-lighter to lighter-heavier. The loads are arranged as shown in Fig. 1.15 (6). 66" 10°41N Give small movement to the left, We Wa _6+6+10_ 10+4 x L-x 15 15 = 147 - 056= +091. Giving small movement to the right, 0.5 —+12.5}e3+4e34254-—— 195. 15 m——2e——25 m: 4 912.5}¢3¢342. 5 25 m 15m——4 6 6 10 10 4kN Wi Wa _6+6 10+10+4 x L-x° 15 40-15 18 — 0.96= — 0.16. aa You have either reached a page that is unavailable for viewing or reached your viewing limit for this book. ROLLING LOADS 33 Taking moment of all loads about load no. 5, we get 92% = (16 x 3) + (20 x 6) + (20 x 10) + (20 x 14) ¥=704 m Let us try with the third load (ée., 20 KN load). Maximum M under it will occurwhen the centre of the span is equidist- ant from load no. 3 and the C.G. of the loads. Distance of load no. 3 and W =7.04 - 6= 1.04 m. Distance of load 3 from centre of span 16KN16KN20 KN 20.KN. 20 KN 1.04 8 = 7052 m 11m——+e 346 3-le 4 males mat (py) ie. load no. 3 is at a distance of 0.52 m from the centre of the span, as shown FIG. 1.16. in Fig. 1.16 (a). In this position, Ry = 7 [ (6 x 5.98) + (16 x 8.98) + (20 x 11.98) + (20 x 15.98) + (20 x 19.98)] =48 kN Meas =+ Ro(12.5 + 0.52) — (20 8) ~ (20 x 4)=+ 48 x 13.02 — 160 — 80 =+ 384.96 kN-m, (6) Maximum S.F. Maximum S.F. value is either Ry or Rp. As the C.G. of the load can approach nearer to B than to A, Ra>Ra for limiting load position. Keep the first load (ie, 20 KN) just to the left of B. Since next load is at 4 m distance, give a movement of 4 m. ‘Thus, W =92 kN; W, = 20 kN d=4m P =toad leaving the span =20 kN From equation 1.33, oF = 48 — + P= 2X4 _ (20-4 20)= 45 - d= - 155. The negative sign shows that the shear force will decrease if the loads are moved. Hence the arrangement of the loads for maximum S.F. will be as shown in Fig. 1.16 (6). Considering the first 20 KN load just to the left of B, we have Re = pe [(16 x 11) + (16 x 14)+(20 x 17) + (20 x 21)-+(20 x 25)]= 66.4 KN Fax =— Rp=— 66.4 KN Example 1.8. A girder, simply supported over a span 20 m, is traversed by moving loads as shown in Fig. 1.17. Determine the maximum B.M at 8 m from the left hand support Solution : Let us try with the second 3 kN load at the section C as shown. Giving slight motion to the left : 1 34 ‘THEORY OF STRUCTURES We Wa _34+34+3 348 | x L-x 8 12 BN SKN SKN SKN kN = 1.125 - 0.91 = + 0.215 Giving slight motion to the right : W._ Wr _3+3 34348 x L-x 8 12 nO. 17 = 0.75 — 1.117 = - 0.42 ~ since Me _ Wa changes sign from +ve to —ve, the maximum will occur when the x L=x loads are arranged as shown. Ra = AEX 1% 4) + BX 10) + Bx 12) +x 14) + 3 x 16)]=9.4 KN Mase =+ (9.4 x 8) — (3X 4) — 3X 2)= + 57.2 kN-m. 1.8, EQUIVALENT UNIFORMLY DISTRIBUTED LOAD A given system of loading crossing a girder can always be replaced by uniformly distributed load, longer than the span, such that bending moment or S.F,, due to this equivalent: static load, every where is atleast equal to that caused by the actual system of moving loads. Such a static load is known as equivalent uniformly distributed load (E.U.D.L). The E.U.D.L. will be different for B.M and S.F, The bending moment diagram for E.U.D.L. will be a parabola symmetrical about the base and must completely envelope the maximum bending moment diagram for the moving loads. Let us now find the E.U.D.L. for the following cases, for B.M. purposes } (a) Single point load. () U.D.L. shorter than span. (©) Two point loads W, and W; at distance d apart (@) E.U.D.L. for Single Point Load : ‘The maximum BM. at the section C, distant x from left support due to a single point load is given by equation 1.3, en) (2) Equating (1) and (2), we get WE on Yt 2 (L-x) L @-x or w’ we(1.34) ‘The same result could be obtained by equating the bending moment at the centre, ie. wil? _ WL 8 4 ROLLINGLOADS 35 or w =2% which is the same as above. (b) EUD.L. for U.D.L. ‘a the, span : The max. BIM. at the centre of the span, due to, U.DLL shorter than the spgn, is given by Mnsx =+ "1 (L-$) where a is the length of the U.DL. se BASS meee oe span, due 0 EUDL. w' is wht 8 Mat" Equating the two, we get or wl =e (1.35) (c) E.U.D.L. for the point loads W, and W; at a distance d apart : The E.U.D.L. for this must be such that the B.M.D. due to this completely envelops Mx, “Mx, and 'M, diagrams. This can be there if the tangent to the curve of B.M. due to EU.D.L at the support is equal to the greater of the tangents to Mx and 'Mx (or °Mxx) diagrams at their corresponding ends. Thus, in Example 1.2, the equation of *Mx is given by Mx y= 7 (136 - 102) ® (atx =0) =4(136)= F (atx = 0) = 7 (136)=85 ofl) The equation of ‘Mx is given by Mx = (0x - 36) (1- 7g) (12254 — 0.625.x* — 36) Dear = 16) = 12.25 ~ 1.25 x 16= ~ 7.75 (2) The minus sign simply shows that the inclination of the tangent is in anticlockwise direction. Greater 2 due to the actual loading = 8.5 ‘The equation of BM. at any point, due to E.UDL. w’ is wr(L—-)L_ wx (6-1) aL 2 ® (atx =0) =8w' Ge @tx = 0) =8w Equating this to the greater of (1) and (2), we get 8w' =85 36 ‘THEORY OF STRUCTURES w'L? _ 85 , 16 x16 8 8 8 ‘The actual absolute Max. B.M. =288 KN-m, as found in example 12. Similarly, the E.U.D.L. on the considerations of max. shear can also be computed. 1.9, COMBINED DEAD AND MOVING LOAD S.F. DIAGRAMS : FOCAL LENGTH Let a girder AB, simply supported over a span L, carry a uniformly distributed dead load w/unit length. Also due to certain system of moving loads, let w” be the E.U.D.L, based on shear considerations. Fig. 1.18(@) shows the S.F.D. due to dead load. Fig. 1.18(b) shows the S.P.D. due to E.U.D.L. At any distance x, S.F. duc to E.U.DLL. is given by ‘This will give max. B.M. = 34 kN-m. wi Fa(-ve) = "5 and Fx (+ve) =EG-a Fig. 1.18(c) shows the combined S.F.D. obtained after superimposing the two diagrams. Thus, by combining +ve SF. of (a) with —ve S.F. of (), we get final shear = ordinate CC. Similarly by combining +ve S.F. of (a) with +ve S.F. of (b), we get final shear = ordinate C1 Cs. Hence in the combined diagram, FIG, AaB the final shear at any point is given by vertical intercepts between dead load SF. and the curves of E.U.D.L. From Fig. 1.18(c), we make the following observations : At point C, SF.=C,C; and C;C3 (both positive) At point P, S.F.=P,P:(=0) and P,P (positive) Al point Q, —S.F.=Q;Q, (negative) and Q:Qs( =0). At point, D, S.F.=D,D, and D,Ds (both negative). From the above, we make the following conclusions : (a) For all section 10 the left of P, the final S.F. is always positive. () For all section to the right of Q, the final S.F. is always negative. (©) For all section between P and Q, the final S.F, is both positive and negative. That is, the SF. changes sign as the load moves over the portion PQ only. Such a portion of the girder, over which the final S.F. changes sign, is called the focal length. If such a girder is of lattice type, counter bracing is needed for this portion. in Fig. 1.18 (¢), thus, PQ is the focal length of the girder. ROLLINGLOADS ” Example 1.9. Calculate the focal length of a girder of 16 m span carrying a dead toad of 3 Nim and EUD.L. of 6 kNim for shear. Solution : (Fig. 1.18) Let Fy =S.F. due 10 dead load, at any section. =S.F due to B.U.D.L. at any section. ‘Then, Fa at Me ret UIE 3 pe 24-3 ea) we 6r _ 3r 2x16 16 ~@) At the point P [Fig 1.18 (¢)], Fa + Fi(—ve)=0 3x Fi(-ve) +24 -31-FF =0 which gives xaA my symmetry, BQ =AP= Focal length = PQ= AB ~ 2 AP= 16 — 2 x 5.85=4.3 m. Example 1.10, Calculate the focal length of a girder of 16 m span, carrying a dead load of 3 kNim and a uniform live load of 2 kNim, 4 m long, travelling from left to right. Solution. (Fig. 1.18) Fant ME -wret POS 310 4-32 wn(l) For r>S m, Fi (-ve) a Fir 2x4 6 (* At the point P (Fig. 1.18 (c)], Fa+ Fi(-ve)=0 a “ +24-3x-05(¢-2) =0 which gives x =AP=7.14 m By symmetry, QB =AP=7.14.m “. Focal length PQ =16-2% 7.14= 1.72 m Example 1.11. Calculate the focal length of the girder of example 1.2 if it also carries a dead load of intensity 3 kNim over the whole span. Solution : For the given girder :L =16m; Wi=4 kN; W,=6KN; d=6m For any section distant x from A, Feat th wes ate _ 3r=2+24-32 wa) (@) For x>6, max. -ve ap duc to live load is given by = Wx — Fieve) = ye Met Hate a). mt ae ad =6x6__5x- - tor. 6x6__ 5x5 185.4 225 .nf2) 38. ‘THEORY OF STRUCTURES At the point P [Fig. 1.18(c)], we have Fa + Fi(—ve) =0 or w-3x- 324225 =0 which gives x =AP= 7.24 m. (®) Again, for x>8<10, we have Fi(t vey = at ME - the ox-@ (1.18) _ [6(6-2) +406-x-6)]_4 45 5 =+ a +85—3e me) For the point Q, we have Fe + Fi(+ ve) =0 += 34 485-2x =0 . | which gives x =AQ= 896 m. Hence focal length = AQ ~ AP= 8.9 ~7.24= 1.72 m PROBLEMS 1. A single rolling toad of 10 KN rolls along a girder of 20 m span. Draw the diagrams of maximum B.M. and maximum S.F. positive and negative. What will te the absolute maximum () S.F. and BM ? 2. A uniform load of 1 kN/m, 4 m long crosses a girder of 16 m span. Construct the maximum SF. and B.M. diagrams and calculate values at section 6 m and 8 m from left hand support. 3. ‘Two concentrated rolling ioads of 12 and 6 KN, placed 4.5 m apart, travel along a freely supported girder of 16 m span. Sketch the graphs of maximum shearing force and maximum ai ‘moment and indicate the position and magnitudes of the greater value. 4. A simply supported girder has a span of 40 m. A moving load consisting of a enifrly cistributed load of 1 KNim over a length of 8 m preceded by 2 concentrated load of 6 KN moving at a fixed distance of 2 m in front of the distributed load, crosses the beam. Find (a) the point of the beam at which the greatest bending moment occurs, (b) |the: position ‘of the load where it occurs, (c) the value of the greatest BM. 5. A simply-supported beam is traversed by a train of wheel loads of irregular spacing and unequal ‘weights. State and prove (a) rule giving the train position for the bending moment under a particular oad to have its maximum value, and (b) a rule giving the train position for the bending moment at a given point on the beam to its maximum value. 6. A freely supported gantry girder of effective span L carries a travelling crane with two wheel loads, cach 7 a spacing 2, i pacing being es an Find, om fe pipes, ne map Deng moment induced by the toads. If the spacing a is increased, find the maximum value of @ (in terms of L) for which the maximum bending moment will occur at the centre of the span with only one wheel on the girder. (U.L.) ROLLING LOADS 39 7. A.system of moving loads cross a girder of 36 m span which is simply supported at its ends. The loads and their distances are as follows: ‘Wheel loads (kN) 10 0 02 0 6 Distance between centres (m) 3 45 4 38 Determine (@) The maximum bending moment at the quarter span. (6) The maximum bending moment in the girder. For each case, make a sketch of the girder showing clearly the section where the bending moment ‘occurs and the corresponding position of the loads. 8 The following system of concentrated loads roll from left to right on a span of 15 m, 4 kN load leading : Load 2 6 6 5 4 kN Distance 1S 15 2 1 ‘metres For a section 4 m from the left hand support, determine (a) the maximum bending moment, (6) maximum S.F. 9 The following system of wticel loads crosses a plate girder of 30 m span : Wheel load 8 8 WB 15 kN Distance between centres 45 35 4 m Determine the maximum value of the shearing foree which may be produced at the middle point of the span. Also, find the equivalent uniformly cistributed load which could produce the same ‘maximum bending moment at_midspan. 10. A simply supported beam of span L is crossed by a uniformly distributed load of length m and of total weight JW. If L is greater than m, obtain from first principles an expression for the maximum bending moment at any point at distance a from one support. Hence show that a single point load of W (-x) travelling across the span will give the same maximum moment everywhere along the beam as the above uniformly distributed Ioad. (WL) 11. The following arrangement of axle load is carried by a single bridge girder across a clear span of 30 m, ‘Axle loads 5 5 0 0 10 iN Spacing 25 25 25 2s n Determine the maximum bending moment and maximum S.F. at section distant 10 m from left hhand abutment. The 5 KN load leads, and the system may pass over the bridge from either side. 12 A beam, simply supported over a span L is traversed by a uniformly distributed load of intensity w and length %. Ifthe beam also carries @ dead load, uniformly distributed over the span, of intensity indicate on the diagram the length of the beam for which there is reversal of shear force. ANSWERS LSE: * 10 KN; BM. : +50 kN-m, 2 13.12 kNem ; 14.0 kN-m. aa You have either reached a page that is unavailable for viewing or reached your viewing limit for this book. Influence Lines 2.1. DEFINITION An influence line for any given point or section of a structure is a curve whose ordinates represent to scale the variation of a function, such as shear force, bending moment, deflection, etc. at the point or section as unit load moves across the structure. In other words, an influence line for any given point C on a structure is such a curve that its ordinate at any point D gives the bending moment, shear force or similar quantity at C when a unit load is placed at D. For statically determinate structures, the influence lines for B.M., S.F. or stress are composed of straight lines, while they are curvilinear for statically indeterminate structures. The influence lines are very useful in the speedy determination of the value of a function at the given section under any complex system of loading. These also help to determine, in an easy manner, the disposition of the load system so as 10 cause the maximum value of the function at the section. ‘The difference between a curve of BM. or S.F. (as discussed in the previous chapter) and an influence line of BM. or S.F. must be clearly understood at this stage. The ordinate of a curve of B.M. or SF. gives the value of the BM. or S.F. at the section where the ordinate has been drawn, while in the case of an influence line, the ordinate at any point gives the value of the B.M. or S.F. only at the given section (for which the influence line has been drawn) and not at the point at which the ordinate has been drawn. Also, there is one single B.M. or S.P. curve of the whole beam under the action of a given set or train of loads, while there are infinite number of influence lines, one for each section of the beam, drawn for a unit rolling load. 2.2, INFLUENCE LINE FOR SHEAR FORCE, Let us consider a simply supported beam AB of span L, and construct the influence line for S.F. at section C distant x from the left support. The position of the section is fixed, while the unit load moves from left to right. The problem is to plot the variation of S.F. at the given section C, as the unit load moves along the beam, At any instant, let the unit load be at a distant aZ from the support A. Then, Re=a, and Ra= (1-2). Shear force at C = Fe=- Ry=- a ea) ‘The variation is linear, and is valid for all positions of load between 0 to x from A. ay a ‘THEORY OF STRUCTURES When the load is at A, teal" aL=0, « Fe=0 When the load is at C, a=x/L 9) ‘When the unit load crosses the sec- tion C, aL>x, and hence Fo =+Ra=+ (1-@) ‘Thus, the S.F. changes sign as the unit load crosses the section. The vari- ation is linear, and is valid for all load positions between x to L from A. When the unit load is slightly to the right of CG a@L=x or a@=x/L Fe od ae: = A 3 ple Se x—— a = Cy When the unit load is at support B,aL=L or a=1 0 5 a Fe =+ (1-a) pt none a The complete influence line dia- +L @ gram for the S.F. at C is gives in Fig. 21 (0). Tt a LL. for Ry As per definition, the ordinate —y, 4 (h) at a point gives the SF. at C, due to unit load at the point where the ordinate AIG. 21. yi is measured. Hence if a load W; is acting at that point, and y, is the ordinate of LL. under it, the S.F. at C=—W;y,. Similarly, if a load W, is acting at a certain point, and + y2 is the ordinate of the influence line under the point of application of the load, the S.F. at C will be + W2y2. If W, and W; are acting simultaneously, the S.F. at C= — Wiy, + Wry: Hence if the beam is being acted upon by loads W;, Ws, Ws ..., Wn, and ys Yx ... ye are the corresponding influence line ordinate under them, the SF. at C is Fo = Way + Way + Ways + ..Wayn= E Wy (2) In the above equation, the numerical value of ordinate y is to be substituted! with its proper algebraic sign, ie. +ve if it is of positive diagram and — ve if of negative portion of the influence line diagram. Let us now take the case of U.D.L. (w) of lengin a, placed in the position shown in Fig. 2.1 (©). Let us consider a length da of the load, and the corresponding elementary load bW=w.da. Hence S.F, at C, due to the elementary load dW is oFc =dWy (where y is the influence line ordinate under 5’) INFLUENCE LINES 8 Fc =wéay =wX area of the elementary strip of the LL. diagram [shown Fig. 2.1 ‘al Therefore, the shear force at C, due to total U.D.L. of length a given by Fe =2w (day)=w Eday (2.2) =w x area of LL. diagram under the U.D.L. [shown shaded in Fig. 2.1 @)] Hence the S.F. at C, due to UD.L, of length a is equal to the area of the LL. diagram under the U.D.L. multiplied by the intensity of the load. Fig. 2.1 (€) shows the U.D.L. extending to both the sides of the section C. In this case, the S.F. at C is obtained by multiplying the net area by the intensity of the load. Thus, Fe =w(~a, +42) where a, =area of the negative S.F. diagram under the U.D.L. @, = area of the positive S.F. diagram under the load. If a =a, Fe= 0. Influence Line for the Reactions : If the section C is located at the support B, the value of x=L and hence the ordinate of — ve LL. diagram under B=x/L=L/L= 1. Thus, the LL. for reaction at B=ILL. for shear at C when x = L, and is a triangle having a maximum ordinate of unity under B. However the LL. for reactions at A and B can be plotted independently as under : When the load is at a distance a from 4, Rj When the load is at A, a= 0, When the load is at B, cL=L; or Ry=a=1 and Ry=(1-@)=0. Hence the LL. for Rs consists of a wiangle having zero ordinate at A and unit ordinate at B, Similarly, the LL. for Ra consists of a triangle having unit ordinate at A and zero ordinate at B as shown in Fig. 2.1 (g) and (1) respectively. 23, INFLUENCE LINE FOR BENDING MOMENT Let us now construct the ILL. for BM. at C. When the unit load is at a distance aL from A, such that aLr Mc =+ Ra.t=+ (1-a)x (1) ‘The variation is linear, and is valid for load position distant x to L from A. When the load is at C, aL=x Mc = + ( which is the same as equatfon 2.3. Thus, the LL. diagram for Mc is a triangle having a maximum ordinate of 5: (L x) under the section as shown in Fig 22 (b). If there are two loads W, and W; acting, and ify; and y2 are the influence line ordinates under these loads, we have by definition Mc =+ (My: + Mayr) Hence, if there are number of point loads W;, W2, ... Wa and the corresponding LL. ordinates under them are y1, y2 . yn we have Mc =+ Ways + Wayr t oe Wayne t ZWy wn(2.4) Let there be an U.DLL. of intensity w, and length a, as shown in Fig. 2.2(c). Consider an elementary length da of the load, such that the elementary load 5W=wéa. Let y be the average ordinate under the elementary load. Then the BM. at C due to this elementary load is given by OW. =+ OW. y=+wda.y=+w x area of the elementary strip of the LL. diagram [shown thick in| Fig. 2.2(@)]. Hence the B.M.atC, duc tothe total U.D.L. of length a is Mc =+ Ew (da.y)=+w x arca of LL. diagram under U.D.L. | [shown shaded in Fig. 2.2(d)) (2.5) Thus, the BM. at C, due to U.D.L. of length a is equal to the intensity of load multiplied by the area of LL. diagram under the uniformly distributed load. 2.4, LOAD POSITION FOR MAXIMUM S.F. AT A SECTION In chapter 1 on rolling loads, we have derived the load positions for maximum S.F. at a given section. We will now use the influence line for determination of the position of loads for maximum SF. at the section C. We shall take different loading conditions. 1. Single point load. Let a single point load of magnitude W roll from left to right. Referring to LL. of S.F. at the section C distant x from A (Fig. 2.1 (6)} maximum negative S.F. will occur when the load is just to the left of C, and maximum +ve S.F. will occur when the load is just to the right of C. Thus, Fe (max. — ve) =— wee and Fe (max. + ve) =+ E29, 2. U.D.L. longer than the span. From the LL. for S.F. at C, Fig. 2.1(b), it is clear that the max. —ve S.F. will occur when the span AC is loaded and CB is empty and max. +ve S.F. will occur when the span CB is loaded and AC is empty. INFLUENCE LINES 4s Thus, Fe (max. -ve) = and Fe (max. tye) ew ‘ea wha) 3. U.D.L. shorter than the span. Let the ioe of length @ travel from left to right. From Fig. 2.1(b), maximum —ve S.F. at C will occur when the head of the load reaches C, while maximum +ve SF. will occur when the tail of the load is at C. 4. Several Point Loads. For several point loads, we may use the same criterion, as discussed in the previous chapter. Thus, if a load W; is at the section C, with other loads in appropriate position, and the loads are moved by a distance d such that next load comes over C, the change OFc is given by aFe = 44 wy, If the above expression is negative, it indicates an increase in S.F. and the loads must be permitted to roll to get greater S.F. The procedure must be repeated till the above expression changes sign, which indicates that greatest peak has been passed. 2.5. LOAD POSITION FOR MAXIMUM BM. AT A SECTION Here also, we shall consider all the loading conditions : 1. Single Point Load. Let a single point load W roll from left to right. Since the LL. diagram for BM. at C has the maximum ordinate under C itself [see Fig. 2.2 (b)], maximum B.M. will occur when the load is at C itself. Thus Me (max) = wr (L-x) 2. U.D.L. greater than the span. Refer to Fig. 22 (b). Maximum B.M. will occur when the U.D.L. occupies the whole span. Thus Mc (max) =wX area of LL. diagram=w x} xLEL -1)2 4 b-9 3. U.D.L. Shorter than the span. Let the uniformly distributed load be of length a a, The load has to be arranged, with respect A A, By B to section C, in such a way that the area — of the LL. diagram under the load is max- @) imum, Let the load be arranged in the position as shown in Fig. 2.3, so that the (um shaded area of LL. diagram is maximum. al [ ~~.» Thatis, a small movement of the loading (L-: to the left or right will decrease the area it ~ of the LL. diagram. If a movement is given to the left, ordinate aa, will be decreased FIG, 23. while 5b; will be increased, and the net result will be the decrease in the area of LL diagram. Similarly, if a movement is given to the right, ordinate bb; will be decreased while ordinate aa, will be increased and the net result will be the decrease in the area of the ILL diagram. Evidently, maximum area will be obtained only if the ordinate aa; is equal to ordinate bb. 46 ‘THEORY OF STRUCTURES aa, = (L -x) 4 -AA, and bb) =F (L =x) Since aa, must be equal to bb, for maximum area, LE AAs =} -BB: or AC _AiC or CB ~ CB, 8) which is the same as that derived in chapter 1. Hence the maximum bending moment at a section occurs when the section divides the U.D.L. in the same ratio as it divides the span. Mc (max.) =w x area of ILL. diagram under the load = w [ (aa: Fee F + (cer + ba) S (aa; +c) 5, since aa= bby wa px(L—x) Loa, x(L-x))_ war(b—2) oy _ all a | EF «(QL —a) we(27) 4. Several Point Loads Let the loads be so arranged that W, is the resultant of the loads to the left of the section C, and We is the resultant of the loads to the right of C. Let y; and y; be the ordinates under W, and We respectively. ‘The B.M. at C, for this arrangement is given by Mc = Wy) + Wee yr This will be maximum only if a | small movement dd of the loads either FIG. 24. to the left or to the right, will decrease its value. | Let the loads be given a movement dd to the right, and let the new ordinates under Wi, and Wa be (yi +6y:) and (y2~dy:) respectively. The corresponding change Mc is given by OMe = [Win + oy) + We 2 - O2)] — [Mi -y: + Way] = W,.01 — Wa. n= MX bd — We Edd | exh =x) 4 (We _ We ap iy Bee (29) Me L- can change sign only when a wheel load passes the section C, thus increasing Thus, 3Mc is negative when me % x Le Wa and decreasing W.. Thus, to get maximum bending moment at a section, one af the loads should be placed at the section so that if the load is considered as a part of W, the, expression becomes negative (or changes sign), The value INFLUENCE LINES a ( We We x negative. Example 2.1. Two wheel loads of 16 and 8 KN, at a fixed distance apart of 2 m, cross a beam of 10 m span. Draw the influence line for bending moment and shear force for a point 4m from the left abutment, and find the maximum bending moment and shear force at that point. Solution (Fig, 2.5). (@) Max. BM. at C The LL. for BM. at C distant 4 m from A is shown in Fig. 25 (b). The maximum ordinate under C is positive, but if considered as a part of Wa, the expression { &— The B.M. at Cis maximum when 2Wy is maximum. By inspection, Max Occurs when the loads are as in the position shown. os [fo Ordinate under 16 kN load = 2.4 —— O4KN () 24x4 Ordinate under 8 kN load = =16 FIG. 25. ¢-Mo= (16X2.4) + (8X106)= 51.2. KN-m. (6) Max, S.F. at C The LL. for SF. at a section C distant 4 m from A is shown in Fig 25 (c). ‘The ordinate under C are ~2=-4=-04 nad L=# L~~ 0 ¥ By inspection of the LL., max. S.F. occurs when the 16 kN load is just to the right of C, and the 8 kN load is ahead of it. Ordinate under 16 kN load =+ 0.6. Ordinate under 8 KN load =+ > x 4=-+04. 6 a+ grt 06. Fo =+ [16 x 0.6 + 8 x 0.4]= + 12.8 KN It can be shown that the max. —ve S.F. at C will be lesser than 128 kN. Hence maximum S.F. at C will be 128 KN. Example 2.2 Make neat diagrams of the influence lines jor shearing force and BM. at a section 3 m from one end of a simply supported beam, 12 m long. Use the diagrams to calculate the maximum shearing force and the maximum bending moment at this section due to a uniformly distributed rolling load, 5 m long of 2 kN per meter intensity. Solution. (Fig. 2.6) (@) EL. for BM. The ordinate c;¢2 of the LL. diagram for B.M. at C= ix 25. The LL. for BM. at C is shown in Fig. 2.6(b). Since the U.D.L. is shorter than the span, the load is 10 be so arranged that area of LL. diagram under the load is maximum. 8 ‘THEORY OF STRUCTURES For this condition a;a:=6,b2 Let the tail of the load be at a distance ¢ from C. AC _ A.C 3__¢ cB CBR, 9 or 15-3c From which € ‘The end ordinates are : From equation 26, a, a= 25 x 1.75 = 1.3125 25 bib 9 * 5.25 = 1.3125 Mex APSE 2D x (125 +375) x2 = 1781 kN-m » (b) LL. for SF. The LL. for S.F. at C is shown in Fig. 2.6(c). The ordinates under C are 075| im B=- 025, and + 2=+ 075. Byin- =k int o spection, maximum S.F, at C will occur when the tail of the load is at C. The ordinate under the head of load O75 x41 9 3 Then = Fe =w x (shaded area of LL. under U.D.L.)=2x 5 (0.75 +t 5.42 KN. Example 2.3. A simply supported girder has a span of 25 m. Draw on squared paper the influence line for shearing force at a section 10 m from one end, and using the diagram determine the maximum shearing force due to the passage of a knife-edge load of 5 kN, followed immediately by a uniformly distributed load of 2.4 kN per metre extending over a length of 5 m, The loads ‘may cross in either direction, Solution (Fig. 2.7) ‘The LL. ordinate ce, = For maximum -ve SF., the 5 kN load will be just to the left of C, and the U.D.L. behind or to the left of it. In this position, the ordinate aa; under the tail of the U.D.L. is 10 INFLUENCE LINES 9 Fe (—ve) =- (5x 2) -24(242 3 For maximum + ve SF. at C, the 5 KN load will be just to the right of C, with U.D.L to the right of it. In this position, the ordinate bb; under the tail of the load is 12 155 Fe(+ve) = (5x2) +24(342) 3. bby 9 kN Hence the maximum S.F, at the section is the greater of the two. Its value is, therefore, 9 KN. Example 2.4. Four wheel loads of 6, 4, 8 and 5 kN cross a girder of 20 m span, from left to right followed by U.D.L. of 4 KNim and 4m long with the 6 kN load leading. The spacing between the loads in the same order are 3 m, 2 m and 2m. The head of the UDL. is at 2 m from the last § KN toad Using influence lines, catculae the S.F. and BM. at a section 8 m from the left support when the 4 kN load is at centre of the span. Solution. (@) Bending Moment ‘The ordinate of LL. for B.M. at C 8x 12_ = She a8 When the 4 KN load is at the centre of the beam, the arrangement of the other loads will be as shown in Fig. 2.8 (a). ©. Ordinate under 6 KN load 48 = 18x 7228 Ordinate under 4 kN load 48 =48x1=4 Ordinate under 5 kN load 48 ag % 6-36 FIG. 28. Ordinate under head of UD.L. x4=2.4 Mc =+ ZMy=+ [(6X2.8)+ (4x4) +(8x4.8)-+(5x3.6) + (x 24 x 4 x 4)] =+ 108.4 kNom ) (6) Shear Force The ordinates of LL. for SF. at C are ~ 3 Hence ordinate under 6 kN load = 98 x 7=0.35 ordinate under 4 KN load = aa You have either reached a page that is unavailable for viewing or reached your viewing limit for this book. aa You have either reached a page that is unavailable for viewing or reached your viewing limit for this book. aa You have either reached a page that is unavailable for viewing or reached your viewing limit for this book. INFLUENCE LINES 8 ‘When the load is at B, x=7m, o Rest ‘When the load is at C, x=17 m, oR =0. Thus the LL. for Rs is triangle as shown in Fig. 2.10(c). For maximum Rg, let the tail of the U.D.L. be at x from D, so that shaded area is maximum. The criterion, given by Eq. 26 is AD _ AD DC” DC i eae l4 (3-2 )o Ordinate aay = c= (3 7) 11s or = Zp from which rm 3 17 Ble Rp =2 pus + 14) x 3= 7.65 KN 4. (©) LL. for Reaction at C (Rc) When the unit load is AD, distant x from A, Ra= (1 ~ =| and hence pressure on DBC Fxst Rex 10 4x _ 2x . Re =— FE = - e.Ro= 434) ofA) ‘When the load is at A, x=0 2 Re =0 When the load is at D, x=3m, “ Re =-04 Now, let the load be in DBC, distant x from A. Ra will be zero for the range of load position. Hence taking moments about B, we have Mp = 0= 1% (7-2) + Rex 10 or (5) ‘When the load is at D, Re =~ 04 (as before) When the load is at B, x=7m » Re =0 ‘When the load is at C, x=17 m, o Rest] The LL. for Rc is shown in Fig, 2.10 (4). By inspection, maximum Re will occur when the head of the U.D.L. is at C. In this position, ordinate under the tail of UDL. =+ 1 x7=+07 Ro =F +07)3=+ A KN (ie 5.1 KN 4) @ LL. for SF. at a section just to the right of B When the load is between A to B, Fs=— Ro, and hence the variation of Fy will be similar to that of Rc but in the reverse direction. Hence LL. for Fs will have zero ordinate under A and B, and ordinate of + 0.4 under D. When the load is in BC, at distance x from A, Ra=0 and Rp= 17-5 (From Eq. 3) 34 ‘THEORY OF STRUCTURES Hence Fo=+Re=+(17-% | (6) When the load is just to the right of B,x=7 m0. Fy=t (47 ag)=* L When the load is atC, 9 x =17 m Fy=t(17-t)=0. | ‘The complete LL. diagram is shown in Fig. 2.10 (¢). It must be noted that the S.F. is always positive at this section. By inspection, maximum S.F. will occur when the tail of the load is at B. In this position, the ordinate under the head of the U.D.L. = + x7=0,7. 10 Fs (max) =+ 3 (1 +07) x 3=+ 5.1 KN. (© EL. for B.M. at E, 1 m to the right of B When the unit load is in AD, distant x from A, Rex 4 (from equation 4 above) a | My =— Rex 9= ~ FE x 9=— 12x | (2) ‘When the load is A, x= Me =0 | When the 4oad is at D, x=3m Mp 2-36 KN-m. When the load is in DE, distant x from A, Re = (07-75) {4 (from equation 5 above)! x Mz =— Re X 9= — (07- ia} 9= -6.3+09x (8) When the load is at D, x=3m, cs Me =-36 KN-m When the load is at B, x=7m, Mz =0 When the load is at E, x=8m, Mp =+09 KN-m When the load is in EC, distant x from A, Ra=O0 and Rs = (.7- a)" (from equation 3 above) et & | Mz =+ Rex 1=+ (17-55 | When the load is at E, x=8m, o Mp =+09 KN-m. | When the load is at C x=i7m, * Mg =0 | The complete LL. for Me is shown in Fig, 2.10 (f). For maximum Me, let) the tail of the U.D.L. be at x from D. The corresponding area of LL. diagram is shown shaded. Using criterion of equation 2.6, we have AD _AD | DB ~ DB, | 3x i 9 or Gay’ fom which r= 5 m | INFLUENCE LINES 5s ; 3.6 9 Ordinate aa; = bbi= = (3 - 5 |= 2.06 Mz =~ 2 (2.06 + 36)(3)=— 1698. kN-m Example 2.7. Draw dimensioned influence lines for the reactions at A and C and for the bending moment at E, the mid-point of the lower beam CF of the simply supported beam system shown in Fig. 2.11. By the use of these influence lines, calculate the greatest value of Ra,Rc and Mz due to the passage of two 10 kN rolling loads, 2 m apart which travel across the upper beam AB. Solution (@) LL for reaction at A te m2 m1 Let the unit load be in AD, 4, z £ distant x from A. e mn fers) fe-2 mo 6 mf - (a) When the load is at 4, q ymo ‘ =i . hye LLL. for Ry, FO) When the load is at D, F x =6 m, 3 i ©) Ra =0. LL. for Re foamy Let the load be in DB, distant Re x from A, Then ¥ 1 e° @ Ra a2 TL. for Me 6 x =-(§-1) -@ FIG. 211. When the load is at D, = + =6 m, Re =O When the load is at B, x=8m ‘The LL. for Ry is shown in Fig. 2.11(5). By inspection, maximum R, will be obtained when one 10 KN load is at A and other ahead of it at 2 m Ordinate under next 10 load = 2x 423 Ry = (10x 1) + (10 x2 +Ba+ 16.67 kN () IL. for reaction at C ‘When the load is at a distance x from 4, Ro= of) ‘Thus, for the lower beam CF, the downward load at D aa You have either reached a page that is unavailable for viewing or reached your viewing limit for this book. aa You have either reached a page that is unavailable for viewing or reached your viewing limit for this book. aa You have either reached a page that is unavailable for viewing or reached your viewing limit for this book. aa You have either reached a page that is unavailable for viewing or reached your viewing limit for this book. 60 ‘THEORY OF'STRUCTURES 3.2. INFLUENCE LINE OF S.F. FOR GIRDER WITH FLOOR BEAMS. Fig. 3.2(@), shows such a system in which the loads are transmitted to the girder at definite points. For a given position of unit load, the S.F. for the whole of the panel, situated between the nodal points, is constant. Hence LL. is plotied for SF. of a panel and nor for S.F. at a particular section of the girder. Let such a girder or frame consist of n panels, each of length d such that the total length L= nd. Let us plot the LL. for SF. in a panel CD (Fig. 3.2(b)}. Let there be m panels to the left of CD, and (n= m=~1) panels to its right. Thus CD is (m+ 1)th panel from the left support Let the unit load roll from A to G. (a) Load in AC When the load is at A, Ro=0, Feo 0 . md_m When the load is at C, Ro rr aay ' Fon =~ Ro=-™ ” (6) Load in DG | Now, let the load be in portion DG. | When the load is at D, Ry = Gama Yd noma | nd " Foo = + Raz +2=B 1 When the load is at G, Ra =0 (©) Load in CD When the load is at a distance x from C, load transmitted at the panel point C d-x and load transmitted at the panel point D= 5 =-Ro+h eo = ~Ro +4 ‘The variation is linear. When the load is at C, x=0 Feo =~, as before. INFLUENCE LINES FOR GIRDERS WITH FLOOR BEAMS 61 When the load is at D, red. ‘Thus the ordinate cc, =2 and ordinate dd;= + The ordinate is zero at some point O between C and D. The LL. for Foo is shown in Fig. 3.2(c). 3.3. LOAD POSITIONS FOR MAXIMUM S.F. Let us now determine the load positions for maximum S.F. in panel CD. (1) Single Point Load : Maximum —ve shear will occur when the point load is at C and maximum -+ve shear will occur when the load is at D. Wm Fen (~ ve max) =" [3.1(@)] and Fen (+ ve max.) = Fa -m-1) -f3-1Q)] 2) U.D.L. greater than the Span Maximum — ve S.F. will occur when ao is fully loaded and og is empty, and maximum + ve S.F. will occur when og is fully loaded and ao is empty, o being the point of zero ordinate of the LLL. for Fen. The position of the point o can very easily be located by the consideration of the triangles ccjo and dd; 0. Thus, cc, _co_ co da, ~ do cd —co Since cci,dds, and cd are known, co can be calculated, and hence o can be located. (3) U.D.L. shorter than the span (Fig. 3.3) Let a U.D.L. of length a travel from left to right such that a are increased and ordinate y is decreased. However, the decrease of ys increases the ~ ve S.F. Hence, the change in SF. is given by OF co = Ws dx tan 0 — W,dx tan 9 + W, dx tan 8. Sco = (Hy + Ww) tan - Wetange tM ah — a) = Pekan Ws = z GL #£ G3) (where W is the total load). Hence, for the maximum, t - % (or —W,} should change sign. In the limiting case, when the loads are very near, = Hence the maximum S.F. in a panel occurs when the load in that panel is equal to the load divided by the number of panels. 3.4. INFLUENCE LINE OF BM. FOR GIRDER WITH FLOOR BEAMS Let us now draw the ILL. for bending moment at a point P, distant x from ft in the panel CD. . When the unit load is in AC, Mp =+ Ro (L—») (2) Again when the unit load is in DG, Mp =+ Rax (2) Both these variations are same as for a girder without floor beam. It must be remembered that x is a fixed quantity in the above equation. When the load is at C, Ro =a ™ ond 7m = Ordinate c= te) (3) INFLUENCE LINES FOR GIRDERS WITH FLOOR BEAMS 6 When the load is at D, _(i-m-1)d_n-m- Ry = Gamay nama (4) My = Ordinate dd;= If the girder were without the floor beam, the ordinate pp under the section would have been a (L —x), and the corresponding or- dinates cc, and dd, would have been x 1 cess EL x) xp x md Z@-2) (which is the same as found above in Eq. 3) ; and a2a- ome ddy= F (Lx) x(n—m=1)d 1 (-x) nom ~ n (which is the same as found above in Eq. 4). Hence the portion ac; and bd; of LL. ‘for a girder with floor beams can be ‘obtained by constructing the ILL. for the beam assuming it to be without floor beam, making thecentralordinate FIGs mz7E-2) and joining p: to a and b, as shown in Fig. 3.4 (b). To plot the portion of LLL. diagram under the bay CD, consider the unit load at a distance a from C. The panel point load transferred to C and D will be 4=4 4 and 4 respectively. Here a + das (3) This isa linear functionofa. When the load is at C, @=0 and hence Mp= cc, (G)zeee Similarly, when the load is at D, a=d, and Mp= dd (G)= aa Hence the LL. portion under panel CD is obtained by joining c; and d; by a straight line. The figure acid;b is thus the complete ILL. diagram for B.M. at point P is the panel CD. ‘The LL. for the point P in any other panel can also be found in a similar manner. However, when the point P coincides with some panel or node point, such as C, the LL. diagram will be a triangle. Fig. 3.4 (d) shows the LL. for BM. at C. The ordinate cc, under C (Fig. 3.4(d)] md(n—m)d_ mm) _ = = =26-na=4 = meme a -m)d. When n=6 and m=2, cer= 2(6-2d= 3d. 4 ‘THEORY OF STRUCTURES 3.5. LOAD POSITIONS FOR MAXIMUM B.M. t 1. Single Point Load From the inspection of the LL. for Mp, it is clear that Mymax will be obtained by putting the load either at C or at D, depending upon whether ordinate cc; is bigger or dd; is bigger. Thus Moma, =+ 7 (1-2 34 (a) or . =+ Winama Dy 34 (0) 2, U.D.L. longer than the Span Maximum B.M. will evidently occur when the load occupies the whole span. In that case, My=w xshaded area of LLL. diagram. 3. Irregular Load System Let W; be the resultant of the loads to the right of D, Ws the resultant of the loads to the left of C, and W; the resultant of the loads on the panel CD. Let the section P be at a distance b from C (such that b=x— md). Then, it can be proved that the maximum B.M. at P occurs when the expression WE-x)_ {™ + Wy} changes: sign. L As a rule, b= $ so that the above criterion reduces to | wenn Be Wi) changes sign. 3.5) Hence, to get the maximum value of Mp, the procedure is as follows : Place the load on the span such that the span is fully covered (if the load system is long) and with one load at D. First consider this load as part of W: and then as part of W. If this eauses the expression of equation 3.5 to change sign, the position is the maximum required. If not, move the load on until another load comes at C or D, and apply the above criterion again. Example 3.1. A Pratt girder shown in Fig. 3.5 consists of eight panels each 3.5|\m square, the loading being on the lower boom. Draw the influence line for in the member EC and determine the maximum tension and maximum compression in EC due to (@) @ concentrated rolling load of 20 kN. (®) a uniform live load of 10 kNim and 10 m tong. Indicate clearly for each of the four required values the corresponding load positions. Solution. Ifwe pass asection 1.1, it is clear that force in EC is equal to shear in BC x sec 45* = Foc x ¥2. Hence Pac = Fac V2. ) (ly Also, when the load is in ao, S.F. in panel BC will be negative, and hence force in EC will be compressive. Similarly, when the load is in od, S.F. in panel BC will be positive, and hence force in EC will be tensile. Let us first plot the LL. for Fac. Here m=2,n=8; d=35Sm INFLUENCE LINES FOR GIRDERS WITH FLOOR BEAMS 65 Ordinate Ordinate If o is the point of zero S.F., we have bo _co_botco__35 bb, cer bby Fey LYS a*3 bo = 4x bm 4x tal m and co =35-1=25 m (@) Concentrated load of 20 kN Maximum - ve S.F. in BC will occur when the point load is at B, while maximum + ve S.F. will occur when the point load is at C. Hence Fac (—ve max) =]x20= 5 kN nc Fac V2=5 V2 KN (compressive) and Foe (-+ve max)=3x20= 12.5 KN Pees Foc V2= 12.5 V2 KN (tensile) (6) ULD.L. of 10 kNim, 10 m long bo =1 m, (found above) ao =7+1=8 m and od= 28 -8= 20 m. Since the length of U.D.L. is more than ao, max. —ve S.F. will occur when the load occupies the whole of the portion ao. Then Foc (— ve max) =w x Area ob,a =10x2 i = 10x 5x 8x 4510 kN FIG. 33. Pec = Foc V2= 10 V2 kN (compressive) For maximum positive shear, the load should be so arranged that the area of the LL. diagram under it (shown dotted) is maximum. This will happen when ordinate pp.= qq: (ie. point ¢ divides the load pq in the same ratio 2s it divides the base od of the triangle ocd). Hence applying the criterion of equation 2.6, we have (OCs PEL PO. ed cq 10—pe or 25 __pe 20=25 ~ 10—pe From which pe = 1.25 m Also, Hence: 66 ‘THEORY OF STRUCTURES 5,5 _ 5($ +H) x 10x 10= 46.88 EN Puc = Fac .V¥2= 46.88 ¥2= 66.2 KN (tensile) Hence Fc (+-ve max) PROBLEMS 1. A Negirder bridge (Fig. 3.6) has cross-girders at the lower panel points. The diagonals are at 45°, A live joad of 6 KNim (per girder), longer than the span, crosses the bridge, Find the maximum forces in the three members AB, AD, and CD. FIG. 36. ANSWERS: 1 Pa= 126 KN (com.) ; Peo= 96 KN (tensile) | Pap= 46.1 KN (tension). aa You have either reached a page that is unavailable for viewing or reached your viewing limit for this book. 8 ‘THEORY OF STRUCTURES crosses each and every section, and hence the influence line ordinate changes from point to point as the unit load moves along the span. However, in nearly all framed structures and girder with floor beams, the loads (whether knife edge load or uniformly distributed load) are applied at the nodes or joints only, so that we have the case of loads applied at definite points (Fig. 4.1 4,6]. Duc to this, the function such as S.F. and BM. for a pancl, or stress in the member of the panel situated between the nodal points, is constant. Hence influence line is plotted for a panel (or for a member), and not for a particular section of a girder or frame. In this chapter, we shall develop the’ influence lines for stress (or forces) in the members of the following types of trusses/frames. Pratt truss with parallel chords. Pratt truss with inclined chords. Warren truss with parallel chords. Warren trusss with inclined chords. K-truss with parallel chords. Baltimore trus with sub-ties : Through type. Baltimore truss with sub-ties : Deck type. Baltmore truss with sub-struts : Through type Pannsyivania truss with sub-ties. 10. Pannsylvania truss with sub-struts, 11, K-truss with inclined chords. 12. Braced cantilever and suspended span girder. eer aw eee 4.2. PRATT TRUSS WITH PARALLEL CHORDS Fig. 4.2 shows a pratt truss with 6 panels, cach of length 4 m and of ‘height 5 m. Let us draw the influence lines for stresses in members of panels L,L; and L,L3 , The truss is statically determinate. 5 . 5 a nO Teas Vat 4 cos 8 = Tay = 0.625 ; cosecd 78 1.28 (1) Influence line for Pour In order to find stress Py,x, in member U; Us pass a section aa as shown, Evidently: Miz Pou, = —g2 (compression) | where Mj, = bending moment at joint Lr The reason for marking the force Py, v, a8, compressive becomes quite evident by considering the equilibrium of the portion of the truss to the left of section aa. The left portion of the truss is in equilibrium under the action of external forces (ie. Rx and unit load at Ly ) and internal forces in members’ U;U; , UrL2 and L2L3 . Out of these five forces, the lines of action of forces Putz, Piz1 and the unit load pass through point L, where moments of the forces are being taken. Out of the remaining two forces, moment of Ry is clockwise; hence aa You have either reached a page that is unavailable for viewing or reached your viewing limit for this book. 0 THEORY OF STRUCTURES (@) Influence line for P:.1: Pass a section bb. Mi ‘ Pita = “J (tension) Hence the LL. for Piz will be a triangle, having a maximum ordinate of ie xpe3 under L; as shown in Fig, 4.2 (d) | (Influence line for Put; When the unit load is at Ly Pixs =0 When the unit load is at L) Poy, =1 (tension) When unit load is at L> or to the right of Le , Piz; =0 The influence line for Py,,; will therefore be a triangle having a maximum ordinate of unity under L, as shown in Fig. 4.2 (e). (5) Influence line for Pru; ‘The force in LoU; can be found by resolution of forces at A in the vertical direction, When the unit load is at A, R= 1, and hence Prov; =O. When the unit is at Li, 2025 =a7 Ra Pigu, = Ra cosec d= $x 1.28 = 1.07 (comp.) When the toad is at B, Ri=O0 +. Pigs = 0 The LL. for Pier, is shown in Fig. 4.2 (). 4.3. PRATT TRUSS WITH INCLINED CHORDS Fig. 4.3 (@) shows Pratt truss with inclined chords, consisting of 6 panels each of a 4 m length. (1) Influence line for P11) Pass a section aa cutting three members Mu, _ Mi i Puta = ae = =F cension) The influence line diagram will therefore be a triangle having a maximum ordinate 1 (4x 20) _ = 3 (3)

changes sign as the limit load traverses the panel L,L2. 4. LL. for Pez, This is also a diagonal member. Pass a section bb as shown in Fig. 4.4 (a). The vertical component of Pu,., is evidently equal to the S.F. in panel LiL: . 2 Ponty = Firig- 00sec @ = 1.547 Fy, « Thus we find that the numerical value of Py.1, is the same as that of Pysi. . However, the nature of stress is reversed. When the load is at L , Pyzi; is tensile while when the load is at Lz , Pyz2; is compressive. The LL. for Prz1, is shown in Fig. 44 (e). 45. WAREN TRUSS WITH INCLINED CHORDS Fig. 4.5 shows a Warren truss with inclined chords. There are six panels cach of 4 m span. (1) Influence line for Puiu, Pass a section aa to cut members U;U:, UrL; and LyL2. "4 ‘THEORY OF STRUCTURES Mu Puwr = (compression) where x= perpendicular dist- ance of L; from U; U; =OL;sina But ung = 452 = a = 26°34 sina = 0.447 oa = _-2=2m tana OL, =2+4=6m Hence x = OLysina = 6x 0447 268 m Puyur = Mi, / 2.68 (compression) The influence line for Puyuy will be a triangle having a maximum ordinate of i TL for, i 4x 1 _ ‘ Me } x0 y= 124 under Ls i i as shown in Fig. 4.5 (6). i ! (2) Influence line for ba Fos TE Le) Piste Le fOr Puy Pas, = (roms uit = > (tension) FIG. 45 WARREN TRUSS WITH INCLINED CHORDS When the load is at A,Re=0 ; henoe My, and Pri; are zero. ¢ x When the load is at L; , Ratko} Pays 3x 18) =0.75 (tension) When the load is at Li, Ra x6 2 Pry = (3x6) = 1 (tension) When the load is at Ry=1 and Ry=0 Hence My, and Piz. are zero. The influence line diagram for Pz... has zero ordinates under A and B and ordinates of 0.75 and 1.0 under L; and L; as shown in Fig. 4.5 (c). 3. Influence line for Pi u, Mo ee) nie = where —_r = perpendicular distance of O from UzL, = OLising INFLUENCE LINES FOR STRESSES IN FRAMES 8 But OL; =6 m ; sing 0.894 P+ = 6 x 0.894 = 5.36 m Mo Paws = 336 When the unit load is at A, Ry=1; Hence Mo and Pi,v; are 7ero When the unit ‘load is at Li, Ri=+ a = 0833 1 : Pare = x'gg { (1 x6) — (0.833 x 2) }= 0.808 (tension) When the unit load is Ly , Re=+2° = 0.667 4 Pu = xz (0.667 x 2) = 0.248 (compression) Thus, there is reversal of stress in L,U, when the unit load crosses the panel L,L2 . The LL. for Piyu, is shown in Fig. 45 (a). (4) Inflence line for Poss Pass a section 6b to cut members Uj Ur , UiL; and LoLy Mo Pou = where y= perpendicular distance of O from UjL1 = OL,sin@ 2 2 OL, =6 m; sind= 4. = 0.707 ° VPtvE VE =6 x 0.707 = 4.24 m Hence When the unit load is at A, Ry=1, Hence My and Py,., are zero. When the unit load is L, , Ra= a = 0.833 Povey = qoq {0833 2} = 0393 (tension) When the unit load is at B, Ri=0. Hence Mo and Py,1, are zero. ‘The LL. for Puss is shown in Fig. 4.5 (e). 4.6. K-TRUSS Fig. 4.6 shows K-truss consisting of 8 panels of 4 m each. (1) Influence line for Pu.us Pass a section aa as shown in Fig. 4.6 (a). Considering the equilibrium of the portion to the left of section aa and, taking moments about Lz, we get Posuy = “22 (compression) 16 ‘THEORY OF STRUCTURES The LL. for Puyuswill be triangle having a maximum ordinate of 8x24 1 3267! under L; , as shown in Fig. 46 (6) (2) Influence line for Prais Prats = Mes¢ension) The IL. for Prauy will be a trian- gle having a max- imum ordinate of 8x24 1 326! under L>, as shown in Fig. 46 (c). (3) Influence line for Py,v, and Prats MU; and Ma La have the same inclination with the vertical. Hence they will carry equal but opposite stresses. Thus, numerically, Pages™ Pres Pass a section bb and consider the equilibrium of the left portion. Resolv. G8 5epeNES ing vertically, Pryuysind + Puyis sin 6 When the unit load is at A, Ry Pyous and Pyqis are zero. When the unit load is at LR, 2 Pugs sin @ = Fi13=1-0.75=025 (tension) shear in panel Lz L5=Fiziy \ , and hence shear in panel LL; is zero. Therefore, 1x24 | 3p 7 O78 ' or Prous = 25 [au sino=2 | 025 ' = 22 = 0.208 (tension) and Pity =0.208 (compression) INELUBNCE LINES FOR'STRESSES IN FRAMES n When the unit load is at Ly and Puts = 0.52 (tension) When the unit load is at B, Ry=0 . Hence Figrs is zero. Therefore, Puyu, and Pyy1s are zero. The LL, for Pryvy and Pypis ate shown in. Fig. 4.6 (d) and (e) respectively. (4) Influence line for Pig15 Pass a section ce, cutting members U:U; , M:Us , MyLy and L5L, , and consider the equilibrium of the left portion. (@® When the unit load is at L Ret ae =0.75 and Py,y; = 0.208 (tension) Pyzu)Sin@ + Pyyis = 1 — Ry = 10.75 = 0.25 Pyyis = 0.25 — Puyey sin 6 = 0.25 — (0.208 x 0.6)= 0.125 (tension) (i) When the unit load is at Ls Ry = 1220 = 0.605 and Prqus =052 comp.) = Pru 8in@ + Pysis = 1 — Ra = 1 — 0.625 = 0.375 or Pussy = 0375 + (0.52 x 0.6) = 0.687 (tension) (iii) When the unit load is at Ly Ry =1 218 205 and Pryv; = 0.416 (comp.) Prgts =Ra— Prrvssin 9 = 05 — (0.416 x 0.6) = 0.25 (comp.) The LL. for Pyyss is shown in Fig. 4.6. (5) Influence line for Poss Pass a section dd, cutting members U;U, , UsMy, MzL; and L;Ls . Out of these four, stresses in members M2Ls and LL, are known. Consider the equilibrium of the portion to the left of section dd. (#) When the unit load is at L: , Rs =0.75 and Py = 0.208 (comp.) Puyay = 1- Ra — Pups sin 8 = 1 ~ 0.75 ~ (0.208 x 0.6)= 0.125 (comp.) (@) When the unit load is at L; Ry =0.625 and Pyyi3 = 0.52 (tension) Puym) = Ra — Pupis Sin @ = 0.625 — (0.52 x 0.5) = 0.313 (tension) The LL. for Pu; is shown in Fig. 4.6 (g) . The influence lines for stresses in other members can similarly be plotted. 7% ‘THEORY OF STRUCTURES 4.7. BALTIMORE TRUSS WITH SUB-TIES : THROUGH TYPE ‘Simple trusses become uneconomical when the span exceeds 80 to 100 m. Earli¢r, multiple web systems were uscd in long span bridges. However, they are expensive and highly indeterminate. and are no longer used. The mordern trend is to use some form of sub-divided trusses or K-truss. A sub-divided truss is obtained by plac- ing in every panel of the truss some secondary members or diagonals. In contrast with primary members, which are stressed with all posi- tions of the loads, secondary members are stressed only by the loads in certain limited positions. Fig. 4.7 (a) shows a Baltimore truss (through type), with subties. The load moves on the lower chords. (1) Influence tine for Piziy and Pisig Pass a section aa cutting membes U; U, , UrMs, and LaLs « Mu 15 (tension) The influence line will be triangle having a eximunl ordinate 12x60 1 _2 “TR B 3 under L; , as shown in Fig. 4.7 (6). (2) Influence tine for Powe Mu 15 (compression) When the unit load is at Lo 1x60 _5 Maa = 3 FIG. 47, BALTIMORE TRUSS WITH SUD-TIES ; THROUGH TYPE. Przts= Pasig = Purus = INFLUENCE LINES FOR STRESSES IN FRAMES. 9 1 5 3 Puno 275 ($x 2-12) =& =0533 (comp) ‘When the unit load is at Ls , Ry = 1454 2075 TR Pures * 0.75 x 24)= 1.2 (comp.) When the load is at B, Ry =0 Por, = 0 ‘The influence line for Pyju, is shown in Fig. 4.7 (c). (9) Influence line for Pyzr2 Pass a section bb. Consider equilibrium of the lef portion Pon = M%& (tension) My and hence Poy; are zero when the unit load is at A. ‘When the unit load is Lz Puna -ta x 12) = 1 (tension) When the unit load is at L; or beyond Ls on right side, there is no external force to the left of section bb except Rx . Hence My =0 . Therefore, Pyyi, is zero. The LL. for Pui; is, therefore, a triangle having zero ordinates under A and L; and a maximum ordinate of unity under L2, shown in Fig. 4.7 (d). (4) Influence line for Pu;se Considering the equilibrium to the left of section aa, Pur sin@ = shear in panel LyLs = Fats sin@ 13 = 0.781 ; 00 0 = pad Vis Fiz Vie riz When the unit toad is at La Ry = 1% - 0833 Fits = — 1+ Ry = — 1 +0.833 = — 0.167 Finis _ 0.167 sin@~ 0.781 Pusws = 0.214 (compression) When the unit load is at A or G, Frais and Pussy are zero. The LL. for Pura is shown in Fig. 4.7 (). (8) Influence line for Pugs M,Ly is a secondary member. When the unit load is at Lz or to the left of Li Pusi3=0 80 ‘THEORY OF STRUCTURES When the unit load is at L3 , Pyyis= 1 (tension) When the unit load is at Ly or to the right of Ls Pigs; =0 The LL. for Pygis is shown in Fig. 4.7 (/). (6) Influence line for Pysu, Ms Us is a subtie, and is thus a secondary member. Pass a horse shoe section ¢e cutting five members. Out of these, four members pass through L, when produced. Hence take the moments about Ls. Consider the equilibrium of the portion enclosed by the horse shoe section ee. ° When the unit load is at L; or Ly M.,=0, and hence Pryy, = 0. When the unit load is at Ls , we get, by taking moment about point L, Pusu X (15 cos @) = 1 6 (tension) 6 Pas. = T5086 = 0.64 (tension) 15 x 0.625 Thus. LL. for Puy, is a triangle, as shown in Fig. 4.7 (g), and is similar to LL. for Pystx The vertical component of Psu, is equal to 0.64 sin 8 = 0.64 x 0.781 = 05 . Hence it is very interesting to note that in general, for both parallel and non-parallel chord trusses, where the secondary has the same slope as the main diagonal, the vertical components of stress in the secondry diagonal will be equal t0 one half of the load applied at the joint. (6) Influence line for Pysis Pass a section ce cutting four members. Consider the equilibrium of the portion to the left of section ce. Resolving forces vertically, a Pry uy Sin @ + Pys.,8in@ = S.F. in panel Ly L4=Fizcy } The member M;U; will have stress only when the load is in span LoLs (@ When the unit load is at L:, Pigus=0 and Ry = 0.833. 1 Fass, = — 1 + 0.883 = — 0.167; at 34 0.883 = — 0.167; cosec 9 = a5 67 Pasta = Gagy (i) When the unit load is at Ls , Pryu,= 0.64 (tension) and Ry = 0.75 0.214 (compression) 0.25, . Aagi * 064 = 032 (tension) 667 (ii) When the unit load is at Ly , Pyyuy=0 and Ry = O67 = 0.854 (tension) The LL. for Puses is shown in Fig, 4.7 (h). (7) Influence line for Puss Pass a section dd cutting four sections. Consider equilibrium of left portion. Resolving the forces vertically, Puss + Puyvssin@ = shear in panel LeLs= Fras Member M; Us, has suress when the load is in panel L2L, only. Pygus = (0.75 — 1) cosec 0 + Pass = Puss = 0.667 cosec @ = aa You have either reached a page that is unavailable for viewing or reached your viewing limit for this book. 82 ‘THEORY OF STRUCTURES Pury = 1.28 Furw; @ When the unit =~ 1.28 x 0.167=-0214 7A ke. 0.214 (compression) t (i) When the unit load isat U; Ry=3=0.75 and Foyys= 0.75 al tO Up Us uh Tush Fy Us Up Uy Up Up Ug Uny Uz Fusus te KAKI I. P 1.28 Fug N ™é ed waiea ft Pours = 1.28 x 0.75 = 0.96 (tension) Let LL. for Puss is shown in Fig. 48 (d). (4) Influence fine for Poss UsMy is secondary member and hence it will be stressed when the load is in panei U; Us The in- fluence line for Pussn is shown in Fig. 4.8 (e), hav- ing zero ordinate under U; and U, and an ordinate of unity under Us . (5) Influence line for Pras MUL is a secondary member, and hence it will be stressed only when the load is in panel U;Us . Pass a horse shoe section bb cutting four members. Out of these, the line of actin of fres in Be, 4a, BALTIMORE TRUSS WEI SUDTES (OLEH TYE point U; . Hence take the moment of forces about point U; . Consider the equilibrium of the portion enclosed by the section bb. When the unit load is at Uz or Us , Mu; is zero, and hence Prsuy is zero. When the unit load is at Us , we get My, = 1% 6 = Pry; X (12 sin 6) INFLUENCE LINES FOR STRESSES IN FRAMES. a3 Pusey = Seoseco = beoseed = 4 x 1.28 = 0.64 (tension) The LL. for Pau, is shown in Fig. 48 (f). (6) Influence line for Pys14 Pass a section cc cutting four members UsU, , My Uy, My Ls, and LyLs . The stress in M.U, is known. Consider the equilibrium of the left portion. Resolving the forces vertically, we get Pyyu, Sin @ + Pyyi,8in@ = shear in panel Us Us = Fusux In the a above relation, member M;U; will have stress only when the load is in UUs. (@® When the unit load is at Us Pyyy=0 and Ry = 5/6 = 0.833 Pygis = cosee 6. Fyyu, = 1.28 (1 — 0.833) = 0.214 (comp.) (i) When the unit load is at Us, Pagus=0.64 (tension) and Ry = 0.75 Puss = Puses + (0.75 — 1) cosec@ = 0.64 — 0.25 x 1.28 = 0.32 (tension) (ii) When the unit load is at Us, Puyus=0 and Ry = 2/3 Pusis = 3.cosec = 2 x 1.28 = 0.853 (tension) The LL. for Pigs is shown in Fig. 4.8 (g). It will be seen that the influence lines for Porws and Pysu are exactly the same for load position between Uy to U; and between U; 0 Up. (7) Influence line for Py,., Pass a section dd, cutting four members, Considering the equilibrium of the left. portion and resolving the fatces vertically we get, in general. Prgia + Pusu, Sin 6 = shear in panel Us Us = Fusus Member M3 Us will have stress ony when the load is in U2U (), When the unit load is at Us, Ry = 0.833 and Pysc,=0 Pusu, = | — 0.833 = 0.167 (tension) ° (ii) When the unit load is at Us , Ra = 0.75 and Pusu, = 0.64 (tension) Pugis = (0.15 ~ 1) + (0.64 x 0.781) (compression) 0.25 + 0.5 = 0.25 (compression) (iii) When the unit load is at Us, Pysua=0 and Ry =3 Pusey = 3 = 0.667 (compression). The LL. for Pus is shown in Fig. 48 (2). 49. BALTIMORE TRUSS WITH SUB-STRUTS : THROUGH TYPE (1) Influence line for Pusu: Pass a section aa cutting three members. Considering equilibrium of the left portion. Pury = (compression) 84 ‘THEORY OF STRUCTURES The influence tine diagram for Puzuy is thus a triangle having a maximum ordinate of xis x b= 1.067 (comp) under Ly as in Fig. 49 (6). | (2) Influence line for Piziy — My , Puts = Fg (tension) When the unit load is at Ly , Ry 1.833, Puts = e (0.833 « 12) = 0.667 (tension) When the unit load is at Ly , Ra=3¢=0.75 Prat = $ (0.75 x 12 + 1 x 6) = 1 (tension) When the unit load is at L, R, = 48 7 7g (0.667 x 12) = 0.533 (tension) 0.667 Prat = The LL. diagram for Pz,1, is shown in Fig. 4.9 (c). | (3) Influence line for Pryrs Pass section aa. Resolving the forces vertically, | PyoiaSin@ =shear in panel Lsly=Fisig | or Pasig = Fits 00sec 0 15 Viste When the unit load is at L; , Ry = 075 Pusu: = (1 ~ 075) X 1.28 = 0.32 (comp.) When the unit load is at L., Ry = 0.667 Puss = 0.667 x 1.28 = 0853 (tension) ‘The LL. diagram for Pysi« is shown in Fig. 4.9 (A). (4) Influence line for Paycs sin 0.781 ; cosec @ = 1.28 ; cos @ = 0.625 MsLy is a secondary member and hence it will be stressed only when the load is in panel LaL, will be a Evidently, LL. for Pwr, will be a triangle having zero ordinates under L and L, and an ordinate of unity under Ls as shown in Fig 49 ©). | (5) Influence line for Pizus ' Pass a horse shoe section 6b, cutting five members U;Ms , Ms Ls, L1Ms, LyLs and LsLs. Out of these, line of action of forces in four members pass through point L, Hence take the moment of the forces, about Ly and consider the equilibrium of the portion enclosed by the horse shoe section. Note that 12M; is a secondary member, and hence it will be stressed only when the load is in panel L2Ly . When the unit load is at LyMr,=0 and hence Pran= 0 INFLUENCE LINES FOR STRESSES IN FRAMES 85 When the unit load is at Ls , Muy = 1X6=PizaqX(12 sin 6) ‘a age ine = Praus = ayang =} cosecO=3x 1.28=0.64 (comp.) When the unit load is at Ls, Mi,=0 , and hence Pi34,=0 The LL. diagram for Pizay is shown in Fig. 4.9 o- (6 Influence line for Poss Pass a section cc, cutt- ing four members. Con- sidering the equilibrium of the left portion, we get, in general, Puzwy sin @ + Przwysin@ =shear in LiL1=Fizis or Pums + Perms =LLscosec 6= 1.28 Fis1s The member L:M, will be stressed only when in Uz Us. When 1 = 0.833) 1.28 214 (comp) When the unit load is at Ls,Ri=0.75 and Pasay = 0.64 (comp.) Pupny + 0.64 = 0.75 x 128 LL for Phas or Puzary = 0.75 % 1.28 ~ 0.64 .32. (tension) ‘When the unit load is at Ls, Ry =0.667 and |. Hence, TL f0F Pug = 0.835 (tension) The LL. diagram for Parag is shown in Fig. 4.9 @). LL for Py, FIG. $9. BALTIMORE TRUSS WITH SUB-STRUTS : THROUGH TYPE ‘THEORY OF STRUCTURES (7) Influence line for Przzz Pass a section dd, cutting four members U:Mi, UrL2, LaMs_ and LiL;. Out of these, the lines of action of forces in two members pass through A. Hence take the moment of all the forces, about A and consider the equilibrium of the left portion. When the unit load is at A, Pyz1; is evidently zero. When the unit load is at Le, Prswy = 0 1x12 2 = 1 (tension) When the unit load is at Ls, Piss =0.64 (comp.) Penta X 12 = (Proan Sin 8) x 12 Pusiz = 0.64 X 0.781 = 0.5 (tension) When the unit load is at Le Piz =0 Poni X 12 = zero The LL. for Posi: is shown in Fig. 49 (h). 4.10, PENNSYLVANIA OR PEITIT TRUSS WITH SUB-TIES (1) Influence line for Pests or Pass a section aa cutting three members = Mis (compresion where r= perpendicular distance of Ze from Us Us Prolong U,U; backward to meet BA produced in O,. Let @ be the inclination of Ux U, as shown in Fig. 4.10 (a). WS-13 1 ae i tan, = a = & = 0.1667 5 6, = 9° 28' sin OrLe O.A=0-36=54 m Now 7 = O,Losin6; = 90 x 0.165 = 1485 m = ls i Peau = yes (Compression) When the unit load is at Li, Ry= 48 = 0667 -_1 _ = Poss = ig 1.967 x 36 — 1 x 12] = 0.808 (comp.) When the unit load is at Ls, Ry = 22 = 0583 1 Poave = yg [0.583 x 36] = 1.412 (comp.) ‘The LL. diagram for Puju, is shown in Fig. 4.10 (6). 2) Influence line for Pic Muy Z Parts = ss (tension) INFLUENCE LINES FO! The LL. dia- gram for Pres will, therefore, be a triangle having a maximum ordinate of 2x48 1 RB under L, as shown in Fig. 4.10 (c). (3) Influence line for Pusss ‘onsider the equilibrium of the portion to the left Of section aa. Take moment about O, | where two members UpUy and Laks when produced, Moy ¥ whereT= per- pendicuarl distance of UM; from O; O;Lesin 8 inclination of UsLy with O1 Le Poss = 13 77 = 0.734 cosec @ = 1.364 12 008 b= aq = 0.678 ¥ =90x0.734 =66.1 m load is at Le , Ra = 0.667 When the unit IR STRESSES IN FRAMES FIG, 4.10. PENNSYLVANIA OR PETTIT TRUSS (WITH SUB-TIES Pusns = sql 78-0.667%54)= 0.635 (comp.) load is at Ls , Ru = 0.583 1 i. ; Poass = Bey (0-583 x 54) = 0.476 (tension) 88 ‘THEORY OF STRUCTURES The LL. diagram for Puss; is shown in Fig. 4.10 (a). (4) Influence line for Pysis MsLs is a sccondary member, and hence will be stressed only when the unit load is in the panel LiL, . The LL. diagram will be a triangle having zero ordinates under Li and Le, and unit ordinate under Ls as shown in Fig. 4.10 (e). (8) Influence line for Pusi Ms Us is also a secondary member, and will be stressed only when the unit load is in the panel L.L« . Pass a horse shoe section bb cutting five members. Out of these, the lines of action of four forces pass through Ls . Hence take moments about L, and ‘consider the equilibrium of portion enclosed by the horse shoe section, Thus Pusu * (15sing) = Mig 6 sing = —2— = °° Tayae Prgug = ig = Mis Mig sue T5sing 15x 0577 8.65 When the unit load is at Le, Mig=0 and hence Pusu, =0 When the unit joad is at Ls , Mig=1X6=6 My 6 : Pusus = 39s = Beg = 0694 (tension) When the unit load is at Le, Mig=0 and hence Pusyg= 0 The influence line diagram for Pusus is shown in Fig. 4.10 (/). (©) Influence line for Ms Le Pass a section cc cutting four members. Out of these, the line of action |of forces in two members pass through O; . Hence consider the equilibrium of the portion to the left ‘of section cc ind take moments about O, . @® When the unit load is at Li , R¢ = 0677 and Pusu (Pusus8in 8) X O; Le = 1 X Oy La ~ 0667 x OLA [1 78 — 0.667 x 54] = 0.636 (comp.) yn087 5 cos p 1 Puss = 74 x90 (i) When the unit load is at Ls , Ra = 0.583 and Pysi = 0.694 (tension) Taking moments about O: , we get (Pugrg sin 6) O) Le = (1 X OrLs) — (Ra X O1 A) + Pusugsin 6 X 6.5)— (Pus ug C08 X O1 Ls) ‘ gp Ess = a + Paste G79q 39g [C1 % 84) — (0.583 x 54) + (0.694 x 0.577 x 6.5)— (0.694 x 0818 x 84)] 0.113 (compression). When the unit load is at Le, Ry=05 and Pysug=0 (Pusi68in 0) O1Le = Ra x OLA Pasua = Gages = 0.408 (tension) The IL. diagram for Pyszg is shown in Fig. 4.10 @). (7) Influence line for Py,v, : The influence line for Py, can be drawn exactly in the same manner as that for Pysv, . The LL. diagram is a triangle with a central ordinate of 08 as shown in Fig. 4.10 (h). Reader is advised to compute this ordinate. INFLUENCE LINES FOR STRESSES IN FRAMES (8) Influence line for Pri, Pass a section dd cutting four members. If @ is the inclination of member M;U, with the vertical 6 _6 V8s+6e 10. Prolong U,U backwards to meet BA produced in O; . Let 6; be the inclination of Us U; with horizontal. Then ang= Bo2 Sat | 6 = 18°26" sin 6: = 0.316 and cos 6, = 0.949 _ Ul Orly tan 8, 13x3=39 m O,A =39-2%=15m. Take the moments about O;, of all unbalanced forces to the left of the section dd. (© When the unit load is at Le, Re 0.833 and Pusu =0 sina = 0577 ; cosa M 1 i Poste = Gaps = 9g" = Fp { EX 2) ~ (0833 x 15)} = 0372 (tension). (@ When the unit toad is at Ls, Re =0.75 and Pryes = 8 (tension) Prugtg X Orbea = (Pury vq 008 & X O24) — (Puyugsina x 13) + (Ry x O2A)— (1 x OzLs) Posts = 5 (08 x 0818 x 39) — (08 x 0.577 x 13) + (0.75 x 15)= (1% 33) = 0.058 (tension) (i) When the unit toad is at Ly , Ry =0.667 and Pryvs=0 Puss = #5 (1% 39) — (0.067 x 15) } = 0.744 (tension). (jv) When the unit load is at Ls , Ry =0.583 and Pusu, =0 Posts = 3 (0583 X 15) = 0.224 (comp,) ‘The influence line diagram for Pox, is shown in Fig. 4.10 (0. 4.1, PENNSYLVANIA OR PETTIT TRUSS WITH SUB-STRUTS Fig, 4.11 (a) shows Pennsylvania or Pettit truss with sub-struts. The truss consisis of 12 panels, each of 6 m length. Let us draw the influence lines of the members of the main panel Lay. 1. Influence line for Pys1, (and Prgis ) Pass a section aa cutting three members UsUs , MsLe and LsLe . Mux _ Mus Pasta Git = Mes ‘When the unit load is at Ly , Ry = 8/12. wh = Paste = 7 [ qq * 24] = 1-231 (lenston) ‘THEORY OF STRUCTURES When the unit load is at Ls , Ry = 7/12 wl, Piste = 35 75 xXM+1x 6] 1.538 (tension) fs ty be tT bs Pile ‘4 1 dab bic Tt 19.260 LLfor Pu, FIG. 411, PENNSYLVANIA OR PETIT TRUSS WITH SUB-STRUTS. INFLUENCE LINES FOR STRESSES IN FRAMES n When the unit load is at Ls , Ry = 1/2 Lyi Paste =ala* 2} = 0.923 (tension) When the unit load is either at A or at B, Mu, is zero and hence Piys1, is zero. The LL. for Pisis is shown in Fig. 4.11 (b) . Since the stress in LiLs is equal to stress in LsLe this also represents the LL. for Prycs - 2. Influence line for Pu, where x is the perpendicular distance of U,Us from Ls . Prolong UsU, and LeLs to meet at OQ, Let 6, be the inclination of U,U, with horizontal. tan) = 5 1667 ; 0; = 9.462" sin 6; = 0.1644 and cos 6; = 0.9864 _ Usks 15 Now O1Le = Gee. = oiee7 = 8998 m OA’ 1.98 — 36 = 53.98 m Now x =O; Losin 6 = 89.98 x 0.1664 = 14.973 m P, Mug Male = T9973 When the unit load is at Le Ry = 1/2 Puss X 36] = 1.202 (compressive) L 1 (4973 (2 When the unit load is either at A or at B, Mz, is zero and hence Pu.u, is zero. The LL. for Pusu, will therefore be a triangle having zero ordinates at A and B, and an ordiante of 1.202 under Ls , as shown in Fig. 411 (c). 3. Influence line for Pasig : Pasie= = where y is the perpendicular distance of M B for bending moment Mp at the point @ D. According to Miiller-Bresiau princi- ple the internal stress component, for which the influence line is to be plotted, is first removed. For the present case, this is accomplished by inserting a pin at D. The beam will then deflect, under the unit load at X, as shown in Fig. 5.10 (b). Let ¢'px be the rotation at D, due to unit load at X. Now remove the unit load from X, and apply a pair of unit moments at D, as shown in Fig. 5.10 (c). Let ¢vp = rotation at D, due to unit couple at D. y'xo = deflection at X due to unit couple at D. From method of consistent deformation (Chapter 7), we have Mp.po= #'px where Mp = bending moment atD duc to unit load at X. FIG. $10 INFLUENCE LINE FOR Mp p'ox M “=. °* Goo But from reciprocal theorem, ¢'px =)'xo (see Eq. 7.7) y'xp Mp = 2 (5.6) Eq. 5.6 suggests that Mp is proportional to y'xp. In other words, the deflection curve of Fig. 5.10 (c) gives, to some scale, the influence line for Mp. If however, goo is selected to be unity, the deflection at any point X will give the bending moment at D. Eq. 5.6, therefore, further proves the validity of the Miller-Breslau principle applied to the influence line for bending moment. 56. CONTINUOUS BEAM : INFLUENCE LINE FOR SHEAR FORCE Let us now study the applicability of the Maller-Breslau principle for plou- 1 ing the influence line for the shear force A BD B x. m @ at any point D of a continuous beam ABC shown in Fig. 5.11. Let the unit load be at any point X. In order to remove the internal stress components, ie shear Fp at D, assume that the beam is cut at D and that a slide device inserted in such a way that it permits relative transverse deflection between the two parts of the cut, as shown, in Fig. 5.11 (6), but which at the same time, maintains a common slope at both the ends of the cut. FIG. 5.11 INFLUENCE LINE FOR SF. AT D THEORY OF STRUCTURES Now remove the unit load from X, and apply a pair of unit loads (shear) at D. The beam will deflect as shown in 5.11 (c). Let Fo = shear force at D. Yox Ho relative linear deflection at D due to unit load at X. lative linear deflection at D due to pait of unit load (shear) at D. deflection at X due to pair of unit load (shear) at D. ‘we have rien, “from compatibility of deformation at D, we Fo. Yoo = Yox or Fo= Y8% Yoo But from the reciprocal theorem, Yox = Yao = Yo Poa 6D ‘Thus, Fy is proportional to Yxo. In other words, the deflection curve of Fig. 5.11 (c) gives the influence line for shear at D, to some scale, If, however, Yoo is taken as unity, the deflection at any point X of Fig. 5.11 (c) give that the shear at D due to unit vertical load at X. 5.7. INFLUENCE LINE FOR HORIZONTAL REACTION Let us now study the influence line for horizontal reaction at the hinged end A of a frame shown in Fig. 5.12 (a). The unit vertical Joad can travel on BC, or unit horizontal load can travel on AB. Let us first take the case when the unit vertical load travel on BC. According to the Miller-Breslau principle, the reaction component at A is first removed. This is accomplished by supporting end A on rollers. The end A will deflect horizontally by Aax due to unit vertical load at X, as shown in Fig. 5.12 (6). The unit vertical load at X is then removed anda unit horizontal load is applied at A. The frame will deflect as shown in Fig. 5.12 (0). Let 4aa = Horizontal deflection of A due to horizontal unit load at A. Axa = Vertical deflection at X due to unit horizontal load at A Hy = Horizontal reaction at A. Then or Ha 3 5.12 INFLUENCE LINE FOR HORIZONTAL REACTION AT A. ‘THE MULLER.-BRESLAU PRINCIPLE 107 But Ax = Axa ftom reciprocal theorem Hence Hy = oe (5.8) Eq. 58 shows that the deflection curve of BC [Fig. 5.12 (c)] gives the influence line, to some scale for Ha when a unit vertical load moves on BC. Similarly, it can be shown that the deflection curve of AB gives the influence line for Ha when a unit horizontal load moves on AB. Exanmple 53 Draw the influence lines for () reaction at B ; and (i) moment at A for the propped cantilever shown in Fig. 5.13 (a). Compute the ordinates at intervals of 1.25 Solution (@) LL for Ry = x8 From Eq. 5.2, Ra= 238 (1) Tocompute yxs, apply a unit vertical load at B, as shown in Fig. 5.13 (6). A x, 8 At any section X distant x from fl — @ B, we have ' a =-M= 1.x I F®e* +0, Integrating, EL => YE dee = FT (6 ~ ae ry | Substituting these in (1), we get x ont L 3 a= (EE ek Zz FIG. 5.13 108" THEORY OF STRUCTURES or or computed in Table 5.1 TABLE St The LL for Rp is shown in Fig. 5.9 (c) (b) IL. for M, In order to draw the LL for Ma, replace the fixed support at 4 by a pin, as shown in Fig. 5.13 (d). Remove the external unit load and apply a unit couple at 4, as shown in Fig 5.13 (e). Then from Eq. 5.3. My =2%4 ih A (3) where y'xa = vertical deflection at X duc to unit couple at 4 Gan = slope at A due to unit couple at 4, Let R's = Reaction at B, when unit moment is acting at A = it and At Cr L/6 Hence wn) and (ii) At (4) At lS) Substituting these values in (3), we get eX _ be) 3 1(x la-él* | | ‘This is thus the equation of the influence line for Ma. The ordinates are calculated in the tabular form in Table 5.2. My ‘THE MULLER-BRESLAU PRINCIPLE 109 TABLE 52 £ (ms) 0 1.25 25 ais | 5 | 62s 15 P/E 0 ooiss | o1s6 | 053 125 245 424 67 100 My 0 oes | 112 | -161_| -187s | -19 | -163 | -1.025 0 The minus sign shows that the direction of Ma is in reverse direction to that of the unit moment applied at A, ie., Ma acts in anti-clockwise direction. The LL for Ma is shown in Fig. 5.13 (). Example 5.4, Determine the injluence line for Ra for the continuous beam shown in Fig. 5.14, Compute the ordinates at every 1 m interval. Solution Apply a unit vertical load at A, as shown in Fig. 5.14 (c). Then Ry =Y84 Yaa A B c From Fig. 5.10 (c), ® Elconsant gy ————SC~S' Ry =24 and Ro=1t4 b+ 4 m +4» 4 m4} (a) Mc=—1 xx | + Ro @e—4) x | +2@-4) 2 Y= -F 40, | +@-4P and > 2 Ely =-E4Cix+G | +eoe At x=4m, y=0, 4. +O. = gs At x=8 m, y= 8C + C= 64 a= 2 and a= =ps FIG. 5.14. x, Hence z At At ae Ei The values of yxa, for various of x are given in Table 5.3. The term 1 has been omitted El for convenience. aa You have either reached a page that is unavailable for viewing or reached your viewing limit for this book. ‘THE MULLER-BRESLAU PRINCIPLE ° mi order to find y'xo Ge. to determine the deflection curve), we shall use the A 8 > © conjugate beam method. Fig. 5.15 (4) rs ” shows a corresponding conjugate beam constant loaded with + M/EI diagram. Thus, an ee zine wl since Max= +2 kN-m, the loading 1 hyo on the conjugate beam will be triangu- lar, having a maximum intensity of +2 (ce, acting downwards) at B, El being omitted. Since the real beam [Fig 5.15 (c)] has a zero deflection at B, the conjugate beam will have a pin at the corresponding point 2B’ so that the B.M. there, representing deflection at B, is vero. Let us first determine the reaction Ry’, Ro’ and Re! of the conjugate beam. Taking moments about the pin B’ and con- sidering the equilibrium of the left portion, we get 1 £x4x2x 3 esac ia = es sidering the equilibrium of = g¢ 8 LL. for Mp the right portion, we have 2 3 f 2Ro! + AR! =1 x 4x2%4 2 3 HIG, 5.15. or Ro! + 2Re! =$ od) Also, Ral + Ro! + Re! = 3 x8 XI or Rp! + Re! =8-R, =8-$=2 aii) From (i) and (ii), we get Ro! =2t and Rc'= —4, ie. acting (4) Now y’x> of real beam = Mx of conjugate beam and goo = relative change in angle at D = sum of shears in conjugate beam to the right and left of the support D = ‘The calculation of Mx (and hence y’xo) and Mp are done in the tabular form below. (Table 5.4). The uence line diagram for Mp is shown plotted in Fig. 5.15 (€). Alternative Solution. The deflection curve or the value of y'xo can also be determined by the conventional double integration method used in example 5.3. Refer Fig, 5.15 (¢), where Ra= 3 t;Ra=1 Jand Ro=} t. Since there is discontinuity in the beam at the pin at D, we will treat the spans AD and DC separately. nz ‘THEORY OF STRUCTURES Mx =y'x0 (pa) pod) =“ (-$x2) + (3x2«1%3) =-2 (po) # (honda) } ($x) + (Exaxaxd) <0 xa (Boer) (Lead? (a+ (fx 2x12) -% doers (Lendl) 8 o @ For the span AD. Measuring x from the LH. support 4, we get BP ASS | +@-4) woe waar ae =~ Ft | +35 3 and By =- Bt axta|+ Atx =0, ys0 2G $ 64 ; __ 6 Atxedm ys0=-5+4C, 5 Gap } Hence the slope and deflection equations for span AD are wy 4) pansy ag Stgl+ ; eal) a= 244] 4 Goa and Ely = otae|+ 5 | (Hl) - 216 4 244 8. _ 26 | Atx=6 m, Ely wtats 7 DY: nS 4 je tL and (FE lon S+ pte i (W For the span DC. Shear at pin D, just to its right, is equal 10 4 4 (ie. shear at D is equal and opposite to Rc). Measuring x from D, to right a Ere? a ‘THE MULLER-BRESLAU PRINCIPLE 13 and = --% : = -% Atr=0, Ely =-9 6 GHB. GQat5. (IH) 4 _x ox 6 and Bye t= 5+ae+2 (IV) = DY) = Atx =0, (BIZ) 0 +5 Now, goo = relative change in this angle at D =(%) -(%) =] co hea (aon ” (Belo EF 3” tne calculation of y'xo is done in table 5.5. The term + has been omitted for convenience. EI TABLE 5.5. 4 THEORY OF STRUCTURES Example 5.6. Determine the influence line for the shear force at D, the middle point of span BC, of a continuous beam shown in Fig. 5.16 (a). Compute the ordinates at 1 mi interval. ‘Solution In order to plot the influence line for shear force Fp, the shearing resist- A Ei constant B D Cc ance of the beam is first removed by @) inserting a sliding device which permits 4m——+h—2 m—s4e— 2 m the relative movement between the two parts but does not impair the moment resistance. Thus the slide device is such that it maintains the same slope in the distorted beam to either side of the device. The unit load (external) is removed and a pair of the unit loads (unit shear) is applied at D. The beam will then distort as shown in Fig. 5.16 (©. The S.F. at D is given by _ Yeo Fe Yoo“) where Yxo = deflection of beam at X due to unit shear at D. Yop = total relative move- ment at D due to unit shear at D. — x(n) = Conjugate beam 3g s Considering the equilibrium of the a a all lo 7 8 portion DC to the right of the cut, we Ae ze BOS have 8 8 8 wiry TP ss 8 8 Re = 1 and #8 Mp =1X2=2 KN-m ( ) Similarly, considering the equilibr- FIG. 5.16. ium of the portion to the left of the cut, and taking moments about 4, we get ARs (4) =Mo+1x6=2+6=8 +. Re= 2h and Ra =2-1=1¢ The bending moment diagram will be a triangle having a maximum ordinate of 4 kN-m at B. (@) Solution by conjugate beam method Fig. 5.16(¢) shows the conjugate beam loaded with M/EI diagram, Omitting EI for convenience, the loading diagram will also be triangle having maximum ordinate of 4 at B, the load acting downwards. In addition to this loading, an unknown moment load y will also act at D’ of the conjugate beam, to satisfy the condition that the slope at both the sides of the slide real beam is the same. Since the slope at the real beam is represented by the shear of the conjugate beam, the shear just to the right of D' must be equal to the shear to the left of D’. This ‘THE MULLER-BRESLAU PRINCIPLE, us condition is satisfied by the moment acting at D’ of the conjugate beam. There is a pin B’ corresponding to the support B of the real beam. Consider the equilibrium of the portion to the left of hinge B’. 1 Ra =f (d adnan dads Re! = total toad Ra'= (5 x8 x 4) — Again, considering the equilibrium of the portion to the right of pin B’, and taking moment about B', we get nt (fxd rand =x 3) 73 _ = 160 _32_ 18 ( Be S33 Since the bending moment of the conjugate beam represent the deflection of the corresponding point of the real beam, the moment u represents the relative movement of the two parts of the slide. Hence Yoo == 128/3 The calculations of Mx (and hence Yxp) and Fp are done in the tabular form below (Table 56). ‘TARLE 56. ° o 0 (peje Geend) od —— ¢ (-$x2) + (Lx202%3) --4 - 23 - -0058 2 ($23) + (3x3 923) =-35 ~ 238 = -0002 4 (- Sxa)+ (pxaxax$ -0 ° s (Sa (jana) B+ 2 Bx 3 - +0108 + ‘S0eny (-Qxaj+ (jx2x2x} +3Ba+d Bx = +0006 xa} + (Leona) = 2 3 za) 3 xi) + xaxixd = 0 ‘The LL. for Fo is shown in Fig. 5.16(a).

You might also like